You are on page 1of 81

EE Set - 2

General Aptitude :
Q.1 to Q.5 carry one mark each
Q.1 The chairman requested the aggrieved shareholders to ____________him.
(A) bare with (B) bore with (C) bear with (D) bare
Ans. (C)
Sol. ‘bear with’ means to be patient with some one
Q.2 Identify the correct spelling out of the given options :
(A) Managable (B) Manageble (C) Mangaeble (D) Managible
Ans. (B)
Sol. ‘manageable’ is the correct spelling
Q.3 Pick the odd one out in the following :
13, 23, 33, 43, 53
(A) 23 (B) 33 (C) 43 (D) 53
Ans. (B)
Sol. In the group of given numbers, all are the prime numbers but 33 is not since it is divisible by 11
Q.4 R2D2 is a robot. R2D2 can repair aeroplanes. No other robot can repair aeroplanes.
Which of the following can be logically inferred from the above statements?
(A) R2D2 is a robot which can only repair aeroplanes.
(B) R2D2 is the only robot which can repair aeroplanes.
(C) R2D2 is a robot which can repair only aeroplanes.
(D) Only R2D2 is a robot.
Ans. (B)
Sol. The statement ‘No other robot can repair aeroplanes’ means R2D2 is the only robot which can repair
aeroplanes so option (B) is the best inference drawn.
Q.5 If 9 y  6  3 , they y 2  4 y /3 is __________.
(A) 0 (B) +1/3 (C) –1/3 (D) undefined
Ans. (A)
Sol. 9y  6  3

9y  6  3 since x  x
GATE 2016 [EE] Set - 2 2

Case I : 9y  6  3
 y 1
Case II : 9y  6  3
1
 y
3
Substitute y  1
4 y 2 4(1) 4 1
y2  1   1  
3 3 3 3
1
Substitute y 
3
1
4  2
4y  1  3 1 4 1
y2        
3 3 3 9 3
Q.6 to Q.10 carry two marks each
Q.6 The following graph represents the installed capacity for cement production (in tonnes) and the actual
production (in tonnes) of nine cement plants of a cement company. Capacity utilization of a plant is
defined as ratio of actual production of cement to installed capacity. A plant with installed capacity of at
least 200 tonnes is called a large plant and a plant with lesser capacity is called a small plant. The
difference between total production of large plants and small plants, in tonnes is ______.
300 Installed Capacity Actual Production

250
250 230
220
200 200
190 190 190
Capacity/Production (tonnes)

200
180
160 150 160 160 150
150 140
120 120
100
100

50

0
1 2 3 4 5 6 7 8 9
Plant Number
Ans. 120
Sol. Installed capacity  200 tonnes  large plant
Installed capacity < 200 tonnes  small plant
Form given multiple pie chart, the large plants are 1, 4, 8 & 9
Total production of large plants = 160 + 190 + 230 + 190 = 770 tonnes
Total production of small plants = 150 + 160 + 120 + 100+ 120 = 650 tonnes
 The difference between total production of large plants and small plants in tonnes
= 770 – 650 = 120 Ans.
3 GATE 2016 [EE] Set - 2

Q.7 A poll of students appearing for masters in engineering indicated that 60% of the students believed that
mechanical engineering is a profession unsuitable for women. A research study on women with masters
or higher degrees in mechanical engineering found that 99% of such women were successful in their
professions.
Which of the following can be logically inferred from the above paragraph?
(A) Many students have misconceptions regarding various engineering disciplines.
(B) Men with advanced degrees in mechanical engineering believe women are well suited to be
mechanical engineers.
(C) Mechanical engineering is a profession well suited for women with masters or higher degrees in
mechanical engineering.
(D) The number of women pursing higher degrees in mechanical engineering is small.
Ans. (C)
Sol. A poll says that women with masters or higher degrees in mechanical engineers are successful in their
professions. This statement leads to the option ‘C’ which is the best inference drawn.
Q.8 Sourya committee had proposed the establishment of Sourya Institute of Technology (SITs) in line with
Indian Institutes of Technology (IITs) to cater to the technology and industrial needs of a developing
country.
Which of the following can be logical inferred from the above sentence?
Based on the proposal,
(i) In the initial years, SIT students will get degrees from IIT.
(ii) SITs will have a distinct national objective.
(iii) SIT like institutions can only be established in consultation with IIT.
(iv) SITs will serve technological needs of a developing country.
(A) (iii) and (iv) only (B) (i) and (iv) only
(C) (ii) and (iv) only (D) (ii) and (iii) only
Ans. (C)
Sol. Option (i) and (iii) state phrases like ‘in the initial years’ and ‘SIT like institutions can only be
established in consultation with IIT’ cannot be logically inferred so (ii) and (iv) are the best inferences
i.e. option (C)
Q.9 Shaquille O’ Neal is a 60% career free throw shooter, meaning that he successfully makes 60 free
throws out of 100 attempts on average. What is the probability that he will successfully make exactly 6
free throws in 10 attempts?
(A) 0.2508 (B) 0.2816 (C) 0.2934 (D) 0.6000
Ans. (A)
Sol. n  number of attempts = 10
x  free throws successfully = 6
n  x  free throws unsuccessfully
 The probability that he will successfully make exactly 6 free throws in 10 attempts
 nCx p x  q n  x [ q  1  p]
60
p  60%   0.6
100
 10C6  (0.6)6  (0.4) 4
10!
  (0.6)6  (0.4) 4
6!4!
 210  0.046656  0.0256  0.2508
GATE 2016 [EE] Set - 2 4

Q.10 The numeral in the units position of 211870  146127  3424 is _________.
Ans. 7
Sol. Cyclicity of 1 is always 1
Cyclicity of 6 is always 6
Cyclicity of 3 is 31  3
32  9
33  7
34  1
Cyclicity of 2 is 21  2
22  4
23  8
24  6
25  2
Cyclicity of 5 is 5  5
Cyclicity of 7 is 71  7
72  9
73  3
74  1
211870  146127  3424  (1) 2  (6)3  1
 1  6 1  7
 Unit digits = ‘7’ Ans.
Technical Part
Q.1 to Q.25 carry one mark each
Q.1 The output expression for the Karnaugh map shown below is
BC
A 00 01 11 10
0 1 0 0 1
1 1 1 1 1

(A) A  B (B) A  C (C) A  C (D) A  C


Ans. (B)
Sol. Using SOP = Sum of product form
BC
A 00 01 11 10
0 1 0 0 1
1 1 1 1 1

Y  AC
Using POS = Product of sum form
BC
A 00 01 11 10
0 1 0 0 1
1 1 1 1 1

Y  AC
Hence correct option is (B)
5 GATE 2016 [EE] Set - 2

Q.2 The circuit shown below an example of a


R2

+15 V
Vin R1
Vout

-15 V

(A) low pass filter. (B) band pass filter. (C) high pass filter. (D) notch filter.
Ans. (A)
Sol. Given : The circuit shown below
R2

C
+15 V
R1
Vin
Vout

-15 V

At low frequency, capacitor C is open. The circuit will look like as shown below
R2

O.C

R1
Vin
Vout

This is an inverting amplifier with voltage gain


Vout  R2
 Gain  
Vin R1
At high frequency, capacitor C is short. The circuit looks like as shown below
R2

S .C

R1
Vin
Vout
GATE 2016 [EE] Set - 2 6

V  V  0 (Virtual ground concept)


Vout  V  0
Vout
 0
Vin
Gain  0 at low frequency
Gain  0 at higher frequency
This circuit is a low pass filter.
Alternate method
Taking Laplace transform and applying virtual ground concept with both node voltages held same and
zero input current to the op-amp.
R2
1
Cs

+15 V
R1
Vin
O Vout

-15 V
Taking KCL at inverting node, we get
0  Vin 0  Vout 0  Vout
  0
R1 1 R2
Cs
Vin  1 
  Vout  Cs  
R1  R2 
Vout 1
 
Vin  1 
R1   Cs 
 R2 
Vout  R2
 …. (i)
Vin R1 (1  R2Cs )
At low frequency, S  0
Vout  R2

Vin R1
At high frequency, S  
Vout
0
Vin
Hence correct option is (A)
Q.3 The following figure shows the connection of an ideal transformer with primary to secondary turns ratio
of 1:100. The applied primary voltage is 100 V (rms), 50 Hz, AC. The rms value of the current I, in
ampere, is __________.
X L = 10 W R = 80 kW
1:100

I
100 V X C = 40 kW
7 GATE 2016 [EE] Set - 2

Ans. 10 to 10
Sol. The given circuit can be drawn by referring impedance of secondary side to primary side we get,
10 jW
1
10 j W 80 ´ kW
1002
I

1 100 V
100 V - 40 ´ j kW
1002
- 4 jW

100V 100V
I 
 (10 j  4 j )  8  (8  6 j ) 
I  10  36.860 A
So, rms value of I will be 10 A Ans.
dy (t ) 1
Q.4 Consider a causal LTI system characterized by differential equation  y (t )  3 x(t ) . The response
dt 6
t

of the system to the input x(t )  3e u (t ) , where u (t ) denotes the unit step function, is
3

t t
 
(A) 9e 3 u (t ) (B) 9e 6 u (t )
t t t t
   
(C) 9e u (t )  6e u (t )
3 6
(D) 54e u (t )  54e u (t )
6 3

Ans. (D)
Sol. Given : The differential equation
dy (t ) 1
 y (t )  3 x(t )
dt 6
Taking Laplace transform we get
1
sY ( s )  Y ( s )  3  X ( s )
6
3  X (s)
Y (s)  …. (i)
 1
s 
 6
And also x(t )  3e  t /3u (t ) (given)
Taking Laplace
3
X ( s)  …. (ii)
1
s
3
Putting equation (ii) in equation (i) we get
3 3
Y (t )  
 1  1
s  s 
 6  3
GATE 2016 [EE] Set - 2 8

Applying partial section


9 A B
  
 1  1 1 1
 s   s   s  6 s  3
 6  3
Evaluating A and B, we get
 1 9
A   s   Y (s)   54
 6 s
1
1 1
6 
6 3
 1 9
B   s   Y ( s)    54
 3 s
1
1 1
3 
3 6
54 54
Therefore Y (s)  
1 1
s s
6 3
Now taking inverse Laplace of above, we get
y (t )  54 e  t /6u (t )  54 e  t /3u (t ) Ans.
Hence the correct option is (D)
Q.5 Suppose the maximum frequency in a band-limited signal x(t ) is 5 kHz. Then, the maximum frequency
in x(t ) cos (200 t ) , in kHz, is _________.
Ans. 6
Sol. Given : x(t ) is a band - limited signal with maximum frequency 5 kHz
X(f )

f
-5 KHz 5 KHz
1
Now, x(t ) cos(2 1000 t ) 
Fourier transform
  X ( f  1000)  X ( f  1000)
2
1
  X ( f  1kHz)  X ( f  1kHz) 1
2
2
1
Plotting X ( f  1kHz)
2
f
1 -4 KHz 6 KHz
Plotting X ( f  1kHz)
2
1
2

f
-6 KHz 4 KHz
From above frequency response maximum frequency present in x(t ) cos (21000 t ) is 6 kHz. Ans.
9 GATE 2016 [EE] Set - 2

Q.6 Consider the function f ( z )  z  z * where z is a complex variable and z * denotes its complex
conjugate. Which one of the following is TRUE?
(A) f ( z ) is both continuous and analytic.
(B) f ( z ) is continuous but not analytic.
(C) f ( z ) is not continuous but is analytic.
(D) f ( z ) is neither continuous nor analytic.
Ans. (B)
Sol. f ( z )  z  z *  x  iy  x  iy  2 x
f ( z )  2 x is continuous (polynomial)
U  2x V 0
Ux  2 Vy  0

Vx  0 Uy  0

Since U x  Vy

Vx  U y
Therefore it does not follows C.R. equations, so not analytical
 Nowhere analytic, so f ( z ) is continuous but not analytic
Hence the correct option is (B)
Q.7 A 3  3 matrix P is such that, P 3  P . Then the eigenvalues of P are
(A) 1,1,  1 (B) 1, 0.5  j 0.866, 0.5  j 0.866
(C) 1,  0.5  j 0.866,  0.5  j 0.866 (D) 0, 1,  1
Ans. (D)
Sol. If  is eigen value of P then  3 is an eigen value of P 3
 P 3  P (given)
3  
 3    0
  ( 2  1)  0
  0 & ( 2  1)  0
   0,  1,  1
Hence the correct option is (D)
Q.8 The solution of the differential equation, for t  0, y "(t )  2 y '(t )  y (t )  0 with initial condition
y (0)  0 and y '(0)  1 , is ( u (t ) denotes the unit step function),
(A) te  t u (t ) (B) (e  t  te  t ) u (t ) (C) (e  t  te t ) u (t ) (D) e  t u (t )
GATE 2016 [EE] Set - 2 10

Ans. (A)
Sol. Given : The differential equation
y "(t )  2 y '(t )  y (t )  0 …. (i)
With initial condition y (0)  0 and y '(0)  1
Taking Laplace transform of equation (i)
 s 2Y ( s )  sy (0)  y '(0)   2  sY ( s )  y (0)   y ( s )  0

 ( s 2  2 s )Y ( s )  ( s  2) y (0)  y '(0)  Y ( s )  0
 ( s 2  2 s )Y ( s )  ( s  2)  0  1  Y ( s )  0
1 1
 Y (s)  
s  2 s  1 ( s  1) 2
2

1
Y (s)  {taking inverse Laplace}
( s  1) 2
So Y (t )  te  t u (t )
Hence the correct option is (A)
Q.9 The value of the line integral

 (2 xy dx  2 x ydy  dz )
2 2

Along a path joining the origin (0, 0, 0) and the point (1, 1, 1) is
(A) 0 (B) 2 (C) 4 (D) 6
Ans. (B)
Sol. Given : Line integral

 (2 xy dx  2 x ydy  dz )
2 2
(1, 1, 1)
c

Along path joining the origin (0,0,0) and (1,1,1)


Equation of straight line
x0 y 0 z 0 (0, 0, 0)
   t (say)
1 0 1 0 1 0

 x  t, y  t, z  t
Taking derivative on both sides,
 dx  dt , dy  dt , dz  dt
Putting limits t  0 & t  1
t 1

 (2 xy dx  2 x ydy  dz )   (4t  1) dt
2 2 3

c t 0

1
4t 4
  t  11  2
4 0

Hence the correct option is (B)


11 GATE 2016 [EE] Set - 2

Q.10 Let f ( x) be a real, periodic function satisfying f ( x)   f ( x) . The general of its Fourier series
representation would be
(A) f ( x)  a0   k 1 ak cos (kx) (B) f ( x)   k 1 bk sin (kx)
 

(C) f ( x)  a0   k 1 b2 k cos (kx) (D) f ( x)   k 0 a2 k 1 sin (2k  1) x


 

Ans. (B)
Sol. Given : f ( x) be a Real
f ( x)   f ( x) odd periodic function
So, cosine terms will be zero in trigonometric Fourier series

 f ( x)   bk sin(kx)
k 1

Hence correct option is (B)


Q.11 A resistance and a coil are connected in series and supplied from a single phase, 100 V, 50 Hz ac source
as shown in the figure below. The rms values of plausible voltage across the resistance (VR ) and coil
(VC ) respectively, in volts, are

VR

VS VC

(A) 65, 35 (B) 50, 50 (C) 60, 90 (D) 60, 80


Ans. (D)
Sol. Given :
I +
VR VR1 Vs VL1
- Vc
VS VC +
VL1
- VR VR1 I

Vc  VR 1  jVL1
Since resistance and the coil is connected in series
Applying KVL
Thus, Vs  VR  jVc

 Vs  VR2  VC2
Using hit and trial method from given options, we have
Vs  602  802  100 V
Hence the correct option is (D)
GATE 2016 [EE] Set - 2 12

Q.12 The voltage (V) and current (A) across a load are as follows.
v(t )  100sin (t )
i (t )  10sin(t  600 )  2sin(3t )  5sin(5t ) .
The average power consumed by the load, in W, is ________.
Ans. 250 to 250
Sol. Given : v(t )  100sin(t ) (fundamental)
i (t )  10sin (t  600 )  2sin (3t )  5sin (5t )
rd
(fundamental) (3 harmonic) (5th harmonic)
Average power consumed by load (P)
P  Vrms I rms cos()
Where,
  angle between fundamental voltage and fundamental current ( voltage has only
fundamental component)
V I
P  m  m cos()
2 2
100 10
   cos 600
2 2
P  250 W Ans.
Q.13 A power system with two generators is shown in figure below. The system (generators, buses and
transmission lines) is protected by six overcurrent relays R1 to R6 . Assuming a mix of directional and
non-directional relays at appropriate locations, the remote backup relays for R4 are
R1 R2
S1 R5 R6 S2
R3 R4

(A) R1 , R2 (B) R2 , R6 (C) R2 , R5 (D) R1 , R6


Ans. (D)
Sol. Given : Power system network
R1 1 R2
s1 s2
R5 R6
3
R3 2 R4

In given diagram R2 , R4 and R5 are directional over current relays and R1 , R3 and R6 are non directional
over current relays according to relay principles for the fault on line 2, R3 and R4 must be operated. If
R4 is not operated then R1 and R6 will operate because R2 and R5 will carry fault current in opposite
direction to set direction.
 Backup for R4 relay are R1 and R6
Hence the correct option is (D)
13 GATE 2016 [EE] Set - 2

Q.14 A power system has 100 buses including 10 generator buses. For the load flow analysis using Newton-
Raphson method in polar coordinates, the size of the Jacobian is
(A) 189  189 (B) 100  100 (C) 90  90 (D) 180  180
Ans. (A)
Sol. Total number of busses = 100
Generator buses  10  1  9  n
Load buses  90  m
Order of Jacobian matrix  (2m  n)  (2m  n)
 (2  90  9)  (2  90  9)  189 189
Hence correct option is (A)
Q.15 The inductance and capacitance of a 400 kV, three-phase, 50 Hz lossless transmission line are 1.6
mH/km/phase and 10 nF/km/phase respectively. The sending end voltage is maintained at 400 kV. To
maintain a voltage of 400 kV at the receiving end, when the line is delivering 300 MW load, the shunt
compensation required is
(A) capacitive (B) inductive (C) resistive (D) zero
Ans. (B)
Sol. Given : L  1.6 mH/km/phase
C  10 nF /km/phase
L 1.6 mH
Surge impedance of transmission line ( Z s )    400 
C 10 nF
VR2 (400 K ) 2
Surge impedance loading ( SI L )    400 MW
Zs 400
Since load applied in transmission link is 300 MW which is less than surge impedance loading
( SIL  400 MW), then the transmission line will act as capacitive.
So to maintain rated voltage at receiving end shunt inductance is required.
Hence the correct option is (B)
Q.16 A parallel plate capacitor filled with two dielectrics is shown in the figure below. If the electric field in
the region A is 4 kV/cm, the electric field in the region B, in kV/cm, is
A B
er = 1 er = 4 2 cm

(A) 1 (B) 2 (C) 4 (D) 16


Ans. (C)
Sol. Given :
A B
er = 1 er = 4 2 cm

Electric field in the region A is 4 kv/cm


GATE 2016 [EE] Set - 2 14

From above figure it is clear that two capacitors are connected in parallel, so the voltage across them is
same and electric field is also same.
V
 E i.e. potential gradient
d V C1 C2 V

So electric field in the region B is 4 KV/cm


Hence the correct option is (C)
Q.17 A 50 MVA, 10 kV, 50 Hz, star-connected, unloaded three-phase alternator has a synchronous reactance
of 1 p.u. and a sub-transient reactance of 0.2 p.u. If a 3-phase short circuit occurs close to the generator
terminals, the ratio of initial and final values of the sinusoidal component of the short circuit current is
________.
Ans. (5.0)
Sol. Given : X synch  1 pu
X subtransient  0.2 pu
Vprefault
We know, fault current (p.u) =
X (p.u)
 Vprefault  1p.u
1
 fault current (p.u) =
X (p.u)
1 1
I subtransient  I initial    5 pu
X subtransient 0.2
1 1
I synch  I final    1 pu
X synch 1
I intial
 Ratio =  5.0 Ans.
I final
Q.18 Consider a linear time-invariant system with transfer function
1
H (s) 
( s  1)
If the input is cos(t ) and the steady state output is A cos(t  ) , then the value of A is _________.
Ans. 0.707
1
Sol. Given : H (s) 
s 1
Y ( s ) Output
H (s)  
X ( s) Input
Y ( s)  X ( s) H ( s)
And Y ( j)  X ( j) H ( j)
Putting x(t )  cos t and taking inverse Laplace
15 GATE 2016 [EE] Set - 2

1 1
 y (t )  cos t  cos t
j  1 1 j1  1
1
y (t )  cos(t  450 ) …. (i)
2
Steady state output
y (t )  A cos (t  a ) …. (ii)
Comparing equation (i) and (ii) we get
1
A  0.707 Ans.
2
Q.19 three-phase diode bridge rectifier is feeding a constant DC current of 100 A to a highly inductive load.
If three-phase, 415 V, 50 Hz AC source is supplying to this bridge rectifier then the rms value of the
current in each diode, in ampere, is ___________.
Ans. (57.6 to 57.8)
Sol. In 3- diode bridge rectifier each diode conduct for 1200 for one complete cycle
2  /3
1 I0
I D rms 
2 0
I 02 d (t ) 
3
100
  57.73A Ans.
3
Q.20 A buck-boost DC-DC converter, shown in the figure below, is used to convert 24 V battery voltage to 36
V DC voltage to feed a load of 72 W. It is operated at 20 kHz with an inductor of 2 mH and output
capacitor of 1000 µF. All devices are considered to be ideal. The peak voltage across the solid-state
switch (S), in volt, is ____________.

Load
S
24 V 2 mH 36 V

Ans. 60 to 60
Sol. Given : Buck Boost converter

S Load
24 V 2 mH 36 V

Since all the devices are ideal, internal drops are not considered.
When switch is OFF, diode D is ON then
PV

24 V 36V

 Peak voltage across switch  24  36  60 V Ans.


GATE 2016 [EE] Set - 2 16

Q.21 For the network shown in the figure below, the frequency (in rad/s) at which the maximum phase lag
occurs is, ___________.
9W

1W
Vin V0
1F

Ans. 0.316 rad/sec


Sol. Given : The network shown below
9W

1W
Vin Vout
1F

Taking Laplace of the above circuit


9

1
Vin Vout
1
s

According to voltage division rule,


1
1
Vout  s V
1 in
9 1
s
Vout s 1
  T ( s ) (say)
Vin 10 s  1
1  j
T ( j) 
1  j10
Let   T ( j)  tan 1 ()  tan 1 (10 )
For calculating frequency at which maximum phase lag occurs,
d
0
d
1 10
  0
1   1  (10) 2
2

 1  1002  10  102
 902  9
1
   0.316 rad/sec for max
10
  is maximum at   0.316 rad/sec Ans.
17 GATE 2016 [EE] Set - 2

Q.22 The direction of rotation of a single-phase capacitor run induction motor is reversed by
(A) Interchanging the terminals of the AC supply.
(B) Interchanging the terminals of the capacitor.
(C) Interchanging the terminals of the auxiliary winding.
(D) Interchanging the terminals of both the windings.
Ans. (C)
Sol. The direction of rotation of a single phase capacitor run induction motor is reversed by inter changing
the terminals of the auxiliary winding. Interchanging the terminal of auxiliary winding reverses the
direction of rotation of the rotating magnetic field.
Hence correct option is (C)
Q.23 In the circuit shown below, the voltage and current sources are ideal. The voltage (Vout) across the
current source, in volts, is
2W
10 V 5A Vout

(A) 0 (B) 5 (C) 10 (D) 20


Ans. (D)
Sol.
2W 5A

10 V 5A Vout

Applying KVL in the given loop


Vout  5  2  10  0
 Vout  20 V
Hence the correct option is (D)
Q.24 The graph associated with an electrical network has 7 branches and 5 nodes. The number of independent
KCL equations and the number of independent KVL equations, respectively, are
(A) 2 and 5 (B) 5 and 2 (C) 3 and 4 (D) 4 and 3
Ans. (D)
Sol. Given : Number of branch, B7
Number of nodes, N 5
Number of independent KCL equation  N  1  5  1  4
Number of independent KVL equation  B  N  1  7  5  1  3
Hence the correct option is (D)
GATE 2016 [EE] Set - 2 18

Q.25 Two electrodes, whose cross-sectional view is shown in the figure below, are at the same potential. The
maximum electric field will be at the point

A D C

B
(A) A (B) B (C) C (D) D
Ans. (D)

Q.26 to Q.55 carry two marks each


Q.26 The Boolean expression (a  b  c  d )  (b  c ) simplifies to
(A) 1 (B) a.b (C) a.b (D) 0
Ans. (D)
Sol. Given : F  (a  b  c  d )  (b  c )
Using De morgan’s theorem i.e. A  B  A  B or A  B  A  B
F  (a  b  c  d )  (b  c )
  (a  b  c  d ) (b  c)
 0
F 0
Hence the correct option is (D)
Alternate method
By simplification
F  ab bcc d
F  a  d 1 ( x  x  0 and 1  x  1 )
F 10
Hence the correct option is (D)
Q.27 For the circuit shown below, taking the op-amp as ideal, the output voltage Vout in terms of the input
voltages V1, V2 and V3 is
9W

+ VCC
1W
V3
1W Vout
V1

V2 4W
-VSS

(A) 1.8V1  7.2V2  V3 (B) 2V1  8V2  9V3


(C) 7.2V1  1.8V2  V3 (D) 8V1  2V2  9V3
19 GATE 2016 [EE] Set - 2

Ans. (D)
Sol. Given :
9W

+ VCC
1W
V3
1W Vout
V1

V2 4W
-VSS

Applying super position theorem


Case - I V2  0, V3  0
9W

1W

V-
Vout = V01
4W V+
V1

4W

4
V  V  V1 
4 1
4
 V  V  V1 (By virtual ground concept)
5
Since it is a non-inverting amplifier
 9 4
So, V01   1  V  10  V1
 1 5
 V01  8V1 …. (i)
Case – II, V1  0, V3  0
9W

1W

V02
4W
V2

1W

1 V2
V  V  V2   (By virtual ground concept)
4 5
GATE 2016 [EE] Set - 2 20

Since it is a non-inverting amplifier


 9 V
So, V02  1  V  10  2
 1 5
 V02  2V2 …. (ii)
Case – III, V1  0, V2  0
9

1W
V3

V03
4W

4W

Since it is a inverting amplifier


9
V03   V3
1
 V03   9V3 …. (iii)
From equation (i), (ii) and (iii) we get
V0  V01  V02  V03
V0  8V1  2V2  9V3
Hence the correct option is (D)
Alternate method
Given
9W

VCC
1W
V3 V-
1W V+ Vout
V1

V2 4W
-VSS

Applying KCL at V
V  V1 V  V2
 0
1 4
5 V
 V  V1  2
4 4
4V1 V2
 V   …. (1)
5 5
21 GATE 2016 [EE] Set - 2

Applying KCL at V
V  V3 V  Vout
 0
1 9
10 V
 V  out  V3
9 9
V 9V
 V  out  3 …. (2)
10 10
Since V  V (By virtual short ground concept)
Vout 9V3 4 V
   V1  2
10 10 5 5
 Vout  9V3  8V1  2V2
Vout  8V1  2V2  9V3
Hence the correct option is (D)
Q.28 Let x1 (t )  X 2 () be two signals whose Fourier transforms are as shown in the figure below. In the
2 t
figure h(t )  e denotes the impulse response.
X 1 (w) X 2 (w)

w w
- B1 - B1 B1 B1 - B2 B2
2 2
x1 (t )

´ h(t ) = e
-2 t y (t )

x2 (t )
For the system shown above, the minimum sampling rate required to sample y (t ) , so that y (t ) can be
uniquely reconstructed from its samples, is
(A) 2B1 (B) 2( B1  B2 ) (C) 4( B1  B2 ) (D) 
Ans. (B)
Sol. From given figure
Bandwidth of X 1 ()  B1
Bandwidth of X 2 ()  B2
y (t )   x1 (t )  x2 (t )   h(t )
1
 X 1 ()  X 2 ()  H ()
Y () 
2
1
Thus the maximum frequency  x1 ()  x2 ()  is band limited with B1  B2 ,
2
Therefore, the minimum sampling rate is 2( B1  B2 )
Hence the correct option is (B)
GATE 2016 [EE] Set - 2 22


 sin 2t 
Q.29 The value of the integral 2    dt is equal to
 
t 
(A) 0 (B) 0.5 (C) 1 (D) 2
Ans. (D)
Sol. Given :

 sin 2t 
I 2   dt
 
t 

2  sin 2t 
   t 
   dt

2

 sin 2t    sin(2t ) 
  2   dt   dt  
 0 t   0 t 2

4 
  2
 2
Hence the correct option (D)
d2y dy
Q.30 Let y ( x) be the solution of the differential equation 2
 4  4 y  0 with initial conditions y (0)  0
dx dx
dy
and  1 . Then the value of y (1) is ___________.
dx x 0
Ans. 7.38 to 7.39
Sol. Given : The differential equation
d2y dy
2
 4  4y  0
dx dx
dy
y (0)  0 and
dx x0

Taking Laplace transformer


s 2Y ( s )  sY (0)  Y '(0)  4 sY ( s )  4Y (0)  4Y ( s )  0

 Y ( s )  s 2  4s  4   1  0

1 1
 Y (s)  
s  4 s  4 ( s  2) 2
2

Taking inverse Laplace


y (t )  te 2t

y (1)  1  e 2  7.389 Ans.


23 GATE 2016 [EE] Set - 2

Q.31 The line integral of the vector field F  5 xziˆ  (3 x 2  2 y ) ˆj  x 2 zkˆ along a path from (0, 0, 0) to (1, 1, 1)
parameterized by ( t , t 2 , t ) is __________.
Ans. 4.41 to 4.42
Sol. Given : F  5 xziˆ  (3 x 2  2 y ) ˆj  x 2 zkˆ
Putting x  t, y  t2, z t
Taking derivatives on both sides
dx  dt , dy  2tdt , dz  dt
Taking line integral

 F  dl   5 xzdx   (3x  2 y )dy   x 2 zdz


2

Putting limits as given


1 1 1
  5 xzdx   (3 x 2  2 y ) dy   x zdz
2

x 0 y 0 z 0

Putting values in terms of t


1
  5t  (3t 2  2t 2 )2t  t 3 )  dt
2

t 0

1
  (5t  10t 3  t 3 ) dt
2

t 0

1
  (5t  11t 3 ) dt
2

t 0

1
5t 3 11t 4
 
3 4 0

5 11 53
    4.41 Ans.
3 4 12
3 1   x a  x
Q.32 Let P    . Consider the set S of all vectors   such that a 2  b 2  1 where    P   . Then S
1 3  y b  y
is
(A) A circle of radius 10
1
(B) A circle of radius
10
1
(C) An ellipse with major axis along  
1
 1
(D) An ellipse with minor axis along  
 1
GATE 2016 [EE] Set - 2 24

Ans. (D)
3 1 
Sol. Given : P 
1 3
 a  3 1   x 
And   b   1 3  y 
    
a   x
Since b   P  y 
   
Now  3x  y  a …. (1)
 x  3y  b …. (2)
Squaring equation (1) and (2) and adding, we get
a 2  b 2  ax 2  y 2  6 xy  x 2  9 y 2  6 xy
 a 2  b 2  10 x 2  10 y 2  12 xy
1 (Given)
 10 x  10 y  12 xy  1
2 2

 a  10, b  10, h  6
h 2  ab  0 i.e h 2  ab  36  100   64
It represents an ellipse
The length of semi-axis are (ab  h 2 )r 4  (a  b)r 2  1  0
64r 4  20r 2  1  0
1 1
r 4  (or) r 2 
4 16
1 1
r  (or)
2 4
2
Since both r values are positive so it represents an ellipse
Length of major axis  2r  1
1
And length of minor axis  2r 
2
 1
Equation of the major axis is  a  2  x  hy  0
 r1 
(10  4) x  6 y  0
 x y 0
And equation of minor axis is
 1
 a  2  x  hy  0
 r2 
(10  16) x  6 y  0
 yx0
 Major axis exists along y   x and minor axis exists along y  x
1
And the vector   lies on the line y  x
1
Hence the correct option is (D)
25 GATE 2016 [EE] Set - 2

Q.33 Let the probability density function of a variable, X, be given as :


3
f X ( x)  e 3 x u ( x)  ae 4 xu ( x)
2
Where u ( x) is the unit step function.
Then the value of ‘a’ and Prob { X  0} , respectively, are
1 1 1 1
(A) 2, (B) 4, (C) 2, (D) 4,
2 2 4 4
Ans. (A)
3
Sol. Given : f X ( x)  e 3 x u ( x)  ae 4 xu ( x)
2
0
For a, 

f ( x).dx  1

 
3
 ae dx   e  3 x dx  1
4x

 0
2

a 1
  1
4 2
 a2 Ans.
0
P ( x  0)   ae
4x
dx


0
 2  e 4 x dx


0
 e4 x 
 2 
 4  
1 1
 2  Ans.
4 2
 1
  2, 
 2
Hence the correct option is (A)
Q.34 The driving point input impedance seen from the source VS of the circuit shown below, in  ,
is_______.
V1 2W
2W
VS 3W 4V1 4 W1
GATE 2016 [EE] Set - 2 26

Ans. 20 to 20
Sol. Given :
+ V1 - 2W
Is 2W

VS 3W 4V1 4W

Z in

Vs
Z in   Driving input impedance
Is
Resistance 2 and 4 are in series, therefore
V1
+ -
Is 2W

VS 3W 4V1 6 W = (2 + 4) W

Resistance 3  and 6  are in parallel, therefore


V1
+ -
Is 2W
æ 3´ 6 ö
2W (3 6 ) = ç
VS 4V1 ÷W
è 3+ 6 ø

V1
Now Is  , therefore figure becomes
2
2W 9I s
Is

VS 8I s 2W

Applying KVL in the above loop


 Vs  2 I s  9  2 I s  0
Vs
Z in   18  2
Is
Z in  20  Ans.
27 GATE 2016 [EE] Set - 2

Q.35 The z-parameters of the two port network shown in the figure are z11  40  , z12  60  , z21  80  and
z22  100  . The average power delivered to RL  20  , in watts, is ___________.
10 W I1 I2

20V V1 [Z ] V2 RL

Ans. (35.5 to 35.6)


Sol. Given : Z11  40 , Z12  60 , Z 21  80 , Z 22  100 
10 W I1 I2

20V V1 [Z ] V2 RL

Using KVL equation,


V1  40 I1  60 I 2 ….(i)
V2  80 I1  100 I 2 ….(ii)
Also 20  10I1  V1 ….(iii)
V2   20 I 2 ….(iv)
Putting equation (iii) in (i)
20 10 I1  40 I1  60 I 2
 50 I1  60 I 2  20
 5 I1  6 I 2  2 ….(A)
Putting equation (iv) in (ii)
 20 I 2  80 I1  100 I 2
 2 I1  3I 2  0 ….(B)
Solving equation (A) and (B) we get
4
I2  A
3
So the average power delivered to ( RL  20)  I 22  RL
2
 4
   20
 3 
 35.55 W Ans.
Q.36 In the balanced 3-phase, 50 Hz, circuit shown below, the value of inductance (L) is 10 mH. The value of
the capacitance (C) for which all the line currents are zero, in millifarads, is ___________.

L L
C

C C
L
GATE 2016 [EE] Set - 2 28

Ans. (3 to 3)
Sol. Given :

L L
C

C C
L
Converting internal Y into  we get

L L
C
C
3 3
C
3

L
For line current to be zero, the parallel pair of inductor capacitor must be in resonant at f  50 Hz
Then XC  X L
1
  L
c
3
1 3
C 2 
 L (250)  10  10 3
2

3
C  3.03 mF Ans.
Q.37 In the circuit shown below, the initial capacitor voltage is 4 V. Switch S1 is closed at t = 0. The charge
(in µC) lost by the capacitor from t  25 s to t  100 s is ____________.
S1

4V
5W
5 mF

Ans. (6.9 to 7.1)


Sol.
S1

4V
5W
5 mF
29 GATE 2016 [EE] Set - 2

VC (0 )  4 V
VC ()  0 V
Time constant   Rc  5  5  10 6  25  sec
t
VC (t )  VC ()  [VC (0)  VC ()] e 
t
VC (t )  4e 
t

VC (t )  4e 2510 6
V
Also q (t )  CV (t )
t t

q (t )  5(4e 2510 6
)  20 e 2510 6
C
6
 2510

At t  25  sec q (t )  20 e 2510 6
 20 e 1  7.357 C
10010 6

At t  100  sec q (t )  20 e 2510 6


 20 e  4  0.3663 C
Total charge lost by capacitor from t  25  sec to t  100  sec
 7.357  0.3663
 6.99 C Ans.
Q.38 The single line diagram of a balanced power system is shown in the figure. The voltage magnitude at the
generator internal bus is constant and 1.0 p.u. The p.u. reactances of different components in the system
are also shown in the figure. The infinite bus voltage magnitude is 1.0 p.u. A three phase fault occurs at
the middle of line 2.
The ratio of the maximum real power that can be transferred during the pre-fault condition to the
maximum real power that can be transferred under the faulted condition is _________.
Generator internal bus Infinite bus
j0.1
j0.5
j0.2 Line 1

j0.5
Line 2
j0.1
Ans. (2.28 to 2.29)
Sol. Given : Single line diagram of power system network in fault bus
Infinite bus
j0.1
j0.5
Vg = 1 pu Line 1
j0.2
V = 1 pu
j0.5
Line 2
j0.1
GATE 2016 [EE] Set - 2 30

Case I : Pre-fault condition


j0.1 j0.5
j0.2
j0.1 j0.5

Vg = 1pu V = 1pu

Transfer impedance
Z1eq  j 0.2  ( j 0.6  j 0.6)

 j 0.2  j 0.3
 j 0.5
Maximum power transfer
Vg  V 1 1
Pmax1    2 pu
Z1eq 0.5

Case II : During fault at midpoint at line 2


j0.1 j0.5
j0.2
j0.1 j0.25 j0.25

Vg = 1pu V = 1pu

j0.6
j0.2
j0.35 j0.25

Vg = 1pu V = 1pu

j0.2 j0.2
d a b

Vg = 1pu j0.35 j0.25 V = 1pu

Converting Ydcb in 
0.2  0.35
X dc  0.2  0.35   0.67 j
0.6
0.35  0.6
X bc  0.35  0.6  2j
0.2
0.2  0.6
X db  0.2  0.6   1.142 j
0.35
31 GATE 2016 [EE] Set - 2

1.142 j

Vg = 1pu j0.67 j2 j0.25 V = 1pu

ß
1.142 j

Vg = 1pu V = 1pu

Vg  V
Maximum power Pmax 2 
X db
1 1
  0.875
1.142
P 2
Now, ratio  max1 
Pmax 2 0.875
 2.2857 Ans.
Q.39 The open loop transfer function of a unity feedback control system is given by
K ( s  1)
G ( s)  , K  0, T  0 .
s (1  Ts )(1  2 s )
The closed loop system will be stable if,
4( K  1) 4(T  2) T 2 8( K  1)
(A) 0  T  (B) 0  K  (C) 0  K  (D) 0  T 
K 1 T 2 T 2 K 1
Ans. (C)
K ( s  1)
Sol. Given : G ( s) 
s(1  Ts ) (1  2s )
K  0, T  0
Characteristic equation is given by
1  G ( s )  0 unity feedback control system
K ( s  1)
1 0
s(1  Ts ) (1  2 s )
s (1  Ts) (1  2s )  K ( s  1)  0
 s (1  2 s  Ts  2Ts 2 )  K ( s  1)  0
 2Ts 3  (2  T ) s 2  s  K ( s  1)  0
 2Ts 3  (2  T ) s 2  (1  K ) s  K  0
GATE 2016 [EE] Set - 2 32

Routh tabulation
2T 1 K 0
s3
2T K 0
s2
(2  T )(1  K )  2TK
s1 0
2T
s0
K
(2  T )(1  K )  2TK
For stability 0
2T
(2  T )(1  K )  2TK
(1  K ) 2T

K 2T
1 2T
  1
K 2 T
1 2T  2  T
 
K 2 T
1 T 2
 
K T 2
T 2
K
T 2
Therefore
T 2
0K 
T 2
Correct option is (C)
Alternately
Characteristic equation
1  G ( s)  0
2Ts 3  (2  T ) s 2  (1  K ) s  K  0
For 3rd order characteristics equation, stability
IP  EP
IP  Internal product
EP  External product
(2  T )(1  K )  2TK
1 K 2T

K 2T
1 2T
  1
K 2 T
1 T 2
 
K T 2
T 2
 K
T 2
Hence correct option is (C)
33 GATE 2016 [EE] Set - 2

Q.40 At no load condition, a 3-phase, 50 Hz, lossless power transmission line has sending-end and receiving-
end voltages of 400 kV and 420 kV respectively. Assuming the velocity of traveling wave to be the
velocity of light, the length of the line, in km, is ____________.
Ans. (294 to 295)
Sol. Due to Ferranti effect VR  VS
 At no load Vs  AVR
400  A  420
400
A  0.9524
420
YZ (r  jL)( g  jC )
We know A  1  1
2 2
For lossless line r  0, g  0 from definition
(C )(L)
Then A  1 …. (i)
2
Also l  2 LC …. (ii)
From equation (i) and (ii) we get,
 2l 2
A  0.9524  1 
2
2l 2  2  0.0476
2l 2  0.0952
l  0.3085
Also we know that
v 2

f 
30  105 2 l
 
50 0.3085
l  294.59 km Ans.
Q.41 The power consumption of an industry is 500 kVA, at 0.8 p.f. lagging. A synchronous motor is added to
raise the power factor of the industry to unity. If the power intake of the motor is 100 kW, the p.f. of the
motor is _____________.
Ans. 0.316
Sol. Given : Power consumption of an industry 500 KVA at 0.8 p.f lagging
P  S cos   500  0.8  400 kW
Q  S sin   500  0.6  300 kVAR
The power factor is to be raised to unity. Qm
S

Thus the motor will supply 300 kVAR. fm


Power intake of motor 100 kW Pm
GATE 2016 [EE] Set - 2 34

Q 
 m  tan 1  m 
 Pm 
Where, Pm  100 kW

Qm  300 kVAR

300
 m  tan 1
100
Qm  71.560

 Power factor of the motor cos m  cos 71.56

 0.316 leading Ans.

( i)
Q.42 The flux linkage (λ) and current (i) relation for an electromagnetic system is   . When i = 2A and
g
g (air-gap length) = 10 cm, the magnitude of mechanical force on the moving part, in N, is ________.
Ans. 282.2

( i)
Sol. Given : 
g

 i  2 g 2
1 di
Force on moving part F   
2 dg
1
     2 2 g
2
1
   3 g  2
2
F  3 g
3
 i
    g
 g 
3
 
 2  10
    282.2 N Ans.
  10   100
  100  
 
Q.43 The starting line current of a 415 V, 3-phase, delta connected induction motor is 120 A, when the rated
voltage is applied to its stator winding. The starting line current at a reduced voltage of 110 V, in
ampere, is _________.
35 GATE 2016 [EE] Set - 2

Ans. 31 to 33
Sol. Given : 415V ,3-,  connected induction motor
( I st )line  120 A at rated voltage
Since all equivalent circuit parameters are constant and frequency is constant therefore current drawn is
proportional to applied voltage
At reduced voltage i.e. V  110 V
( I st )line  x  I st rated
Vreduced
x
Vrated
110
( I st )line at110 V = 120
415
 31.807 A Ans.
Q.44 A single-phase, 2 kVA, 100/200 V transformer is reconnected as an auto-transformer such that its kVA
rating is maximum. The new rating, in kVA, is ______.
Ans. 6 to 6
Sol. Given : Single phase 2 kVA, 100/200 V transformer (two winding)
20 A 10 A
A C
+ +

100 V 200 V

– –
B D
For maximum kVA rating, the output voltage rating as well as output current rating must be largest
possible so the connection shown below
20 A
+
+
100 V
30 A

10 A 300 V
+
+
200 V 200 V

– – –

 (kVA) maximum  maximum possible voltage  maximum possible current


= 300  20  6 kVA Ans.
Q.45 A full-bridge converter supplying an RLE load is shown in figure. The firing angle of the bridge
converter is 1200. The supply voltage vm (t )  200 sin(100t ) V , R = 20 Ω, E = 800 V. The inductor L is
large enough to make the output current I L a smooth dc current. Switches are lossless. The real power
fed back to the source, in kW, is __________.
GATE 2016 [EE] Set - 2 36

IL Load

T1 T3 L

R = 20 W
vm Bridge
E = 800 V
T2
T4

Ans. 6 to 6
Sol. Given :
A full bridge converter with RLE load (i.e. dc motor)
IL

TT11 T3 L

vm
R = 20 W
Load
E = 800 V
TT14 T2

For a full bridge converter with RLE load (i.e. dc motor)


2V
V0  m cos    E  RI L

2  200  cos1200   800  20  I L
I L  30 A
Since switches are lossless
 Power fed back to source  200  30  6 kW Ans.
Q.46 A three-phase Voltage Source Inverter (VSI) as shown in the figure is feeding a delta connected resistive
load of 30 Ω/phase. If it is fed from a 600 V battery, with 1800 conduction of solid-state devices, the
power consumed by the load, in kW, is __________.
30
W

600 V
30

30 W
37 GATE 2016 [EE] Set - 2

Ans. 24 kW
Sol. Given : A three phase voltage source inverter converting delta load into equivalent star
30
R ph   10 
3
In 1800 conduction mode
rms value of each phase voltage
2 2
V ph  Vdc   600  489.89
3 3
V ph2
Power consumed by the load P  3 
R ph
3  (489.89) 2
  24 kW Ans.
10
Q.47 A DC-DC boost converter, as shown in the figure below, is used to boost 360V to 400 V, at a power of 4
kW. All devices are ideal. Considering continuous inductor current, the rms current in the solid state
switch (S), in ampere, is _________.
10 mH

Load
360 V S
1mF 400 V

Ans. 3.5 A
Sol. Given : DC to DC boost converter.
10 mH

Load
360 V S
1mF 400 V

For TON period switch will be ON


TOFF period switch will be OFF
And T  TON  TOFF
For is  switch current

is

TON TON
T
GATE 2016 [EE] Set - 2 38

Vs 1 T
We know for step up chopper  where   ON  duty cycle or duty ratio
Vs 1   T
400 1

360 1  
   0.1
Now Power  Vs I s
4000  360  I s
I s  11.1 A
Neglecting ripple in is
1
 T 2
I switch ( rms )  I s  ON 
 T 
 I s   11.1 0.1  3.5A Ans.
Q.48 A single-phase bi-directional voltage source converter (VSC) is shown in the figure below. All devices
are ideal. It is used to charge a battery at 400 V with power of 5 kW from a source Vs = 220 V (rms), 50
Hz sinusoidal AC mains at unity p.f. If its AC side interfacing inductor is 5 mH and the switches are
operated at 20 kHz, then the phase shift (δ) between AC mains voltage (Vs) and fundamental AC rms
VSC voltage (VC1), in degree, is _________.

5 mH
1mF 400 V IS
IS XS VS
d
220 V AC
VC1
IS X S

Ans. 9.210
Sol. Given : A single phase bidirectional voltage source converter L  5mH, C  1mF
Power from source
P  5 kW IS VS
P  Vs I s ( p. f ) d

5000  220  I s 1 IS X S
VC1
I s  22.72 A
Is X s
From figure tan  
Vs
 22.7  2 50  5  10 3 
  tan 1  
 220 
  9.210 Ans.
39 GATE 2016 [EE] Set - 2

Q.49 Consider a linear invariant system x  Ax , with initial condition x(0) at t  0 . Suppose  and  are
eigenvectors of ( 2  2 ) matrix A corresponding to distinct eigenvalues 1 and  2 respectively. Then the
response x(t ) of the system due to initial condition x(0)   is
(A) e1t  (B) e2t  (C) e2t  (D) e1t   e2t 
Ans. (A)
Sol. Given : x  Ax
Eigen value are 1 and  2
So we have
 e1t 0 
(t )    2t 
0 e 
Response due to initial condition
x(t )  (t ) x(0)
e1t 0   
x(t )    
0 e  2t   0 
x(t )  e1t
Hence correct option is (A)
Q.50 A second-order real system has the following properties:
a) the damping ratio   0.5 and undamped natural frequency n  10 rad/s ,
b) the steady state value of the output, to a unit step input, is 1.02.
The transfer function of the system is
1.02 102 100 102
(A) 2 (B) 2 (C) 2 (D)
s  5s  100 s  10s  100 s  10s  100 s  5s  100
2

Ans. (B)
Sol. Given : Damping ratio   0.5
Undamped natural frequency n  10 rad/sec
Steady state value of the output to a unit step input is 1.02
Transfer function for of 2nd order system is given by
C ( s) k n 2
T (s)  
R ( s ) s 2  2n s  n2
n  0.5 10
n  5 …. (i)
2n  100 ….. (ii)
Now for unit step input
1 k n 2
Output C ( s)   2
s s  2n s  n 2
GATE 2016 [EE] Set - 2 40

Steady state output


C ()  lim C (t )  lim sC ( s )  1.02
t  s 0

k 2n
 lim  1.02
s 0 s 2  2n s  n2
 k  1.02
Hence transfer function of the system using equation (i) and (ii)
1.02 100 102
T (s)   2
s  2  5s  100 s  10 s  100
2

Hence the correct option is (B)


Q.51 Three single-phase transformers are connected to form a delta-star three-phase transformer of 110 kV/
11 kV. The transformer supplies at 11 kV a load of 8 MW at 0.8 p.f. lagging to a nearby plant. Neglect
the transformer losses. The ratio of phase currents in delta side to star side is
(A) 1:10 3 (B) 10 3:1 (C) 1:10 (D) 3:10
Ans. (A)
Sol. Given : Three single phase transformer are connected to form a delta - star there phase transformer.
a
A
V ph
V ph = VL N VL

b c B
C

Ñ side voltage (phase) V ph ( D ) = 110 kV


11
side phase voltage V ph (Y ) = kV
3

\ D side phase voltage Y -side phase current


=
side phase voltage D -side phase current

\ D - side phase current 11


=
- side phase current 3 ´ 110

1

10 3
Hence correct option is (A)
Q.52 The gain at the breakaway point of the root locus of a unity feedback system with open loop transfer
Ks
function G ( s )  is
( s  1)( s  4)
(A) 1 (B) 2 (C) 5 (D) 9
41 GATE 2016 [EE] Set - 2

Ans. (A)
Ks
Sol. Given : G ( s) 
( s  1)( s  4)
img

R-L R-L
Real
s =1 s=4

Characteristic equation
Ks
1 0
( s  1)( s  2)
( s  1)( s  4) ( s 2  5s  4)
K  
s s
For break away points
dk
0
ds
 ( s ) (2 s  5)  ( s 2  5s  4)(1) 
  0
 s2 
 2 s 2  5s  s 2  5s  4 
  0
 s2 
s 4
2

s  2
s  2 will be valid breakaway point K s 2 1
As it lies between two poles.
Hence the correct option is (A)
Q.53 Two identical unloaded generators are connected in parallel as shown in the figure. Both the generators
are having positive, negative and zero sequence impedances of j0.4 p.u., j0.3 p.u. and j0.15 p.u.,
respectively. If the pre-fault voltage is 1 p.u., for a line-to-ground (L-G) fault at the terminals of the
generators, the fault current, in p.u., is ___________.

Ans. 6 to 6
Sol. Given :
GATE 2016 [EE] Set - 2 42

Equivalent circuit for LG fault


Positive sequence diagram

0.4j 0.4j 0.2j


º

Negative sequence diagram

0.3j 0.3j º 0.15j

Zero sequence diagram

0.15j 0.15j
º 0.15j

If
Now for LG fault the sequence network will be as follows, owing to the fact that I a0  I a1  I a2 
3
I a1

0.2 pu

1 pu

I a2 If (Q z f = 0 it is dead short circuit)


0.15 pu

I a0
0.15 pu

V prefault 1
I a1    2pu
X 0  X1  X 2 0.2  0.15  0.15
I f  3I a1  3  2  6 pu Answer.
43 GATE 2016 [EE] Set - 2

Q.54 An energy meter, having meter constant of 1200 revolutions/kWh, makes 20 revolutions in 30 seconds
for a constant load. The load, in kW, is _____________.
Ans. (2 to 2)
Number of revolutions
Sol. Meter constant =  1200(given)
kwh
20
1200 
30
kW 
3600
20  3600
kW   2 kW Ans.
1200  30
Q.55 A rotating conductor of 1 m length is placed in a radially outward (about the z-axis) magnetic flux
density (B) of 1 Tesla as shown in figure below. Conductor is parallel to and at 1 m distance from the z-
axis. The speed of the conductor in r.p.m. required to induce a voltage of 1 V across it, should be
__________.
z

1m
1m

Ans. 9.55
Sol. From electrostatics, definition of potential
1
Voltage induced   Em . dl
0

Where Em  induced electric field


Since voltage induced = 1V (Given)
So Em  1v/m
From definition of Lorentz force
We know that Em  v  B
Where, V  radius of path  angular velocity
1V
 v  1 Telsa
m
v  1m/sec
v  r    1m/sec
Where r  1m
  1rad/sec
GATE 2016 [EE] Set - 2 44

N
Also   2
60
Where N  number of revolutions/min
N
1rad/sec  2
60
60
 N  9.55 rpm Ans.
2


EE Set - 1

General Aptitude :
Q.1 to Q.5 carry one mark each
Q.1 The man who is now Municipal Commissioner worked as ____________________.
(A) the security guard at a university (B) a security guard at the university
(C) a security guard at university (D) the security guard at the university
Ans. (B)
Sol. Option (A) and (D) cannot be the answers because of the word ‘the security’. The word ‘university’ is
always prefixed with ‘the’ being important.
Q.2 Nobody knows how the Indian cricket team is going to cope with the difficult and seamer-friendly
wickets in Australia.
Choose the option which is closest in meaning to the underlined phrase in the above sentence.
(A) put up with (B) put in with (C) put down to (D) put up against
Ans. (A)
Sol. ‘cope with’ means to ‘put up with’, it relates performing
Q.3 Find the odd one in the following group of words.
mock, deride, praise, jeer
(A) mock (B) deride (C) praise (D) jeer
Ans. (C)
Sol. ‘mock, deride and jeer’ are synonyms which means mockery. Therefore, the odd one is ‘praise’
Q.4 Pick the odd one from the following options.
(A) CADBE (B) JHKIL (C) XVYWZ (D) ONPMQ
Ans. (D)
Sol.
(A) 1 2 (B) 8 9

C A D B E J H K I L
(C) 23 24 (D) 13 14

X V Y W Z O N P M Q
In options A, B and C, the letters skipped between consecutive pair of letters in the English alphabet is
in increasing order (i,e) A and B, H and I and V and W but in option ‘D’ N and M are present instead of
M and N so, option ‘D’ is odd one from the group.
GATE 2016 [EE] Set - 1 2

Q.5 In a quadratic function, the value of the product of the roots (α, β) is 4. Find the value of
 n  n
  n   n
(A) n 4 (B) 4n (C) 22 n1 (D) 4n1
Ans. (B)
 n  n  n  n  n  n
Sol.   n   n n
1 1
 n  n  n
n n
  n

 
n
 
 n n  (  ) n  (4) n
Q.6 to Q.10 carry two marks each
Q.6 Among 150 faculty members in an institute, 55 are connected with each other through Facebook® and
85 are connected through WhatsApp®. 30 faculty members do not have Facebook® or WhatsApp®
accounts. The number of faculty members connected only through Facebook® accounts is
______________.
(A) 35 (B) 45 (C) 65 (D) 90
Ans. (A)
Sol. Total faculty members = 150
The faculty members having facebook account = FB = 55
The faculty members having whatsapp = W = 85
The faculty members do not have face book (or) Whats App accounts = 30
The faculty members having any account = 150 – 30 = 120
The faculty members having both the accounts = (FB + W) – 120 = (55 + 85) – 120 = 20
 The number of faculty members connected only through Facebook accounts = 55 – 20 = 35
Facebook

150

35 20 65 WhatsApp

Q.7 Computers were invented for performing only high-end useful computations. However, it is no
understatement that they have taken over our world today. The internet, for example, is ubiquitous.
Many believe that the internet itself is an unintended consequence of the original invention. With the
advent of mobile computing on our phones, a whole new dimension is now enabled. One is left
wondering if all these developments are good or, more importantly, required.
Which of the statement(s) below is/are logically valid and can be inferred from the above paragraph?
(i) The author believes that computers are not good for us.
(ii) Mobile computers and the internet are both intended inventions
(A) (i) only (B) (ii) only (C) both (i) and (ii) (D) neither (i) nor (ii)
Ans. (B)
Sol. The first and second sentences tell us that computers are invented for computation and internet for
intended invention. These sentences lead to option ii so option (B) is the right inference
3 GATE 2016 [EE] Set - 1

Q.8 All hill-stations have a lake. Ooty has two lakes.


Which of the statement(s) below is/are logically valid and can be inferred from the above sentences?
(i) Ooty is not a hill-station.
(ii) No hill-station can have more than one lake.
(A) (i) only (B) (ii) only (C) both (i) and (ii) (D) neither (i) nor (ii)
Ans. (D)
Sol. Statement (i) is not true because Ooty is a hill station and Ooty has two lakes statement(ii) is also not
true, because in given statements, for hill station one lake is compulsory but not mentioned about
number of lakes.

Hill stations
Lake

Q.9 In a 2 × 4 rectangle grid shown below, each cell is a rectangle. How many rectangles can be observed in
the grid?

(A) 21 (B) 27 (C) 30 (D) 36


Ans. (C)
Sol. In given 2  4 rectangle grid, the following type of rectangles are present.
One figured rectangles = 8
Two figured rectangles = 10
Three figured rectangles = 4
Four figured rectangles = 5
Six figured rectangles = 2
Eight figured rectangles = 1
Total No. of rectangles = 30
 The No. of rectangles observed in the given grid = 30.
Q.10
2.5 f ( x)
2.0
1.5
1.0
0.5
x
–4 –3 –2 –1 0 1 2 3 4
–0.5
–1.0
–1.5
–2.0
–2.5
Choose the correct expression for f (x) given in the graph.
(A) ( ) = 1 − | − 1| (B) ( ) = 1 + | − 1| (C) ( ) = 2 − | − 1| (D) ( ) = 2 + | − 1|
GATE 2016 [EE] Set - 1 4

Ans. (C)
Sol. @ x = 3 from given graph f ( x) must be equals to zero
Option (A)
f ( x)  1  x  1
@x 3
f ( x)  1  3  1
 1  2  1
So, it is not
Option (B)
f ( x)  1  x  1
@x 3
f ( x)  1  3  1
 1 2  3
So, it is not
Option (C)
f ( x)  2  x  1
@x 3
f ( x)  2  3  1
 22  0
So, it is true
Option (D)
f ( x)  2  x  1
@x 3
f ( x)  2  3  1
 22  4
So, it is not

Technical Part
Q.1 to Q.25 carry one mark each
Q.1 The maximum value attained by the function f ( x)  x( x  1)( x  2) in the interval [1, 2] is _____.
Ans. 0 to 0
Sol. Given : f ( x)  x( x  1) ( x  2) in the interval [1, 2]
f ( x)  x( x 2  x  2 x  2)
 x3  3x 2  2 x
f '( x)  3 x 2  6 x  2
5 GATE 2016 [EE] Set - 1

For maximum value of f ( x)


f '( x)  0 will give points of maxima and minima
 3x 2  6 x  2  0
x  1.57, 0.422 ( x  0.422 is not in the range [1, 2])
In the range [1, 2] x  1.57
f ''( x)  6 x  6
f ''(1.57)  3.42  0 which is a point of minima
Thus at x  1.57 f ( x) is minimum
For maximum f (1)  0
And f (2)  0
Therefore maximum value attained by f ( x) is 0 Ans.
Q.2 Consider a 3 × 3 matrix with every element being equal to 1. Its only non-zero eigenvalue is ____.
Ans. 3 to 3
1 1 1
Sol. Given : A  1 1 1
1 1 1
1 1 1  0 0 
Now [ A  I ]  1 1 1   0  0 
1 1 1  0 0  
1   1 1 

  1 1  1 
 1 1 1   
1   0 1 

 1  1  c2  c2  c3
 1  1   
Characteristic equation, A  I  0
1   0 1 
  1  1   0

 1  1   
 (1  )[(1  )   ]  (  )  0
 (1   )( 2  2 )  2  0
 [(1   )(  2)  2]  0
  ( 2  3 )  0
  2 (3   )  0
  0,0,3
Hence its non zero eigenvalue is 3 Ans.
GATE 2016 [EE] Set - 1 6

Q.3 The Laplace Transform of f (t )  e 2t sin(5t ) u (t ) is


5 5 s2 5
(A) (B) (C) (D)
s  4 s  29
2
s 52
s  4 s  29
2
s5
Ans. (A)
Sol. Given : f (t )  e 2t sin(5t ) u (t )
a
We know L[sin at u (t )] 
s  a2 2

From property of frequency shifting function


L[e at f (t )]  f ( s  a )
5
Therefore L[sin 5(t ) u (t )] 
s  52
2

5 5 5
L[e 2t sin 5t u (t )]   2  2
( s  2)  5
2 2
s  4  4 s  25 s  4 s  29
Hence the correct option is (A)
Q.4 A function y (t ) , such that y (0)  1 and y (1)  3e 1 , is a solution of the differential equation
d2y dy
2
 2  y  0 . Then y (2) is
dt dt
(A) 5e1 (B) 5e2 (C) 7e 1 (D) 7e2
Ans. (B)
Sol. Given the differential equation
d2y dy
2
2  y 0
dt dt
d
 ( D 2  2 D  1) y  0 where D 
dt
 D2  2D  1  0
( D  1) 2  0
 D  1,  1
 solution y  (k1t  k2 ) e  t
Applying boundary condition to evaluate k1 and k2
Putting t  0, y (0)  1, k2  1
Putting t  1, y (1)  3e 1
3e 1  (k1 1  1) e 1
k1  2
Putting values of k1 and k2
 y (t )  (2t  1) e  t
y (2)  (2  2  1) e  2  5e 2
Hence the correct option is (B)
7 GATE 2016 [EE] Set - 1

Q.5 The value of the integral


2z  5
  1 2
dz
C
 z   ( z  4 z  5)
 2
Over the contour z  1 , taken in the anti-clockwise direction, would be
24i 48i 24 12
(A) (B) (C) (D)
13 13 13 13
Ans. (B)
2z  5
Sol. Given :  
1 2
dz
C
 z   ( z  4 z  5)
 2
Equating denominator to zero so as to find poles
 1 2
 z   ( z  4 z  5)  0
 2
1
z  , 2i are singularities
2
1
Only z  lies inside C because z  1 is continuous and rest all lies beyond it
2
By applying residue theorem
 1
  2  Res at
c
i
2
 
1   1 2z  5  24
 Residue at  lt  z   
2 z  1  2 1 2  13
2  z   ( z  4 z  5)
  2 

 1  24 48i
 2i  Res at    2i 
 2  13 13
Hence correct option is (B)
Y ( s) s
Q.6 The transfer function of a system is  . The steady state output y (t ) is A cos(2t  ) for the
R( s) s  2
input cos (2t ) . The values of A and  , respectively are
1 1
(A) ,  450 (B) ,  450 (C) 2,  450 (D) 2,  450
2 2
Ans. (B)
Y ( s) s
Sol. Given : 
R( s) s  2
r (t )  input = cos(2t )
For y (t ) putting  = 2
GATE 2016 [EE] Set - 1 8

i.e. s  j  2 j
2j
Y ( s ) s 2 j  R( s)
22 j
2j
y (t )  cos 2t
22 j
1
 y (t )  cos(2t  450 ) …. (i)
2
y (t )  A cos (2t  ) …. (ii) (given)
Comparing (i) and (ii) we get,
1
A ,   450 Ans.
2
Hence correct option is (B)
100
Q.7 The phase cross-over frequency of the function G ( s )  in rad/s is
( s  1)3
1
(A) 3 (B) (C) 3 (D) 3 3
3
Ans. (A)
100
Sol. Given : G(s) 
( s  1)3

 G ( j)   3tan 1 
To find phase crossover frequency PC , we have

G ( j)    1800
PC

3tan 1 PC  1800

 PC  3 rad/sec

Hence the correct option is (A)


Q.8 Consider a continuous-time system with input x(t ) and output y (t ) given by
y (t )  x(t ) cos (t )
This system is
(A) linear and time-invariant
(B) non-linear and time-invariant
(C) linear and time-varying
(D) non-linear and time-varying
9 GATE 2016 [EE] Set - 1

Ans. (C)
Sol. Given : y (t )  x(t ) cos (t )
(i) For linear condition
y1 (t )  x1 (t ) cos(t ) …. (i)
y2 (t )  x2 (t ) cos(t ) …. (ii)
Adding above equation (i) and (ii)
y1 (t )  y2 (t )  [ x1 (t )  x2 (t )]cos(t )
Which satisfies additivity
Multiplying y (t ) by k and x(t ) by k
y1 (t )k  x1 (t )k cos t
ky1 (t )  kx1 (t ) cos t
Which satisfies Homogeneity
Therefore given system is linear
(ii) Time varying/invariant
It the input is delayed by t0
y1 (t )  x(t  t0 ) cos t
If the output in delayed by t0
y (t  t0 )  x(t  t0 ) cos (t  t0 )
Hence y1 (t )  y (t  t0 ),So, it is time varying
Hence the correct option is (C)


e
t
Q.9 The value of (2t  2)dt , where (t ) is the Dirac delta function, is


1 2 1 1
(A) (B) (C) (D)
2e e e2 2e 2
Ans. (A)
 

 e  t (2t  2)dt  e
t
Sol. (2(t  1))dt
 


1
e
t
 (t  1)dt

2

1
  e  t (t  1)dt
2 
1 1
 e 1 
2 2e
Hence the correct option is (A)
GATE 2016 [EE] Set - 1 10

Q.10 A temperature in the range of 400 C to 550 C is to be measured with a resolution of 0.10 C . The
minimum number of ADC bits required to get a matching dynamic range of the temperature sensor is
(A) 8 (B) 10 (C) 12 (D) 14
Ans. (B)
Sol. Given : Temperature range   400 C to 550 C
So total range  55  ( 40)
 950 C
Resolution  0.10 C
Range 950C
Number of steps   0  950
Resolution 0.1 C
For minimum number of ADC bits
950  2n
950  210
950  1024
 n  10
The minimum ‘n’ value which can satisfy the above equation is 10
Hence the correct option is (B)
Q.11 Consider the following circuit which uses a 2-to-1 multiplexer as shown in the figure below. The
Boolean expression for output F in terms of A and B is

Y F

1S

A
B

(A) A  B (B) A  B (C) A  B (D) A  B


Ans. (D)
Sol. Given :

Y F

1S

A
B
(i) B  0 FA
(ii) B  1 FA
 F  AB  AB  A  B
 A B Ans.
Hence the correct option is (D)
11 GATE 2016 [EE] Set - 1

Q.12 A transistor circuit is given below. The Zener diode breakdown voltage is 5.3 V as shown. Take base to
emitter voltage drop to be 0.6 V. The value of the current gain β is _________.
10 V

4.7 kW 220 W

0.5 mA

5.3V 470 W

Ans. 19 to 19
Sol. Given :
10 V

I IC

4.7 kW 220 W

0.5 mA
IB +
+ 0.6
-
Vz = 5.3V 470 W
-
IE

Zener diode is in breakdown therefore it is replace by a voltage source Vz  5.3 V


Applying KCL at base we get
 5.3V  0.6 V  I E  0.47 k  0
I E  10 mA …. (i)
Applying KCL through supply, 4.7 k resistor and zener diode
10  I  4.7  5.3  0
I  1 mA …. (ii)
From figure
I B  I  0.5 mA  1  0.5 mA
I B  0.5 mA …. (iii)
From figure and equation (iii)
I C  I E  I B  10 mA  0.5 mA
I C  9.5 mA …. (iv)
I C 9.5
Current gain     19
I B 0.5
Hence   19 Ans.
GATE 2016 [EE] Set - 1 12

Q.13 In cylindrical coordinate system, the potential produced by a uniform ring charge is given by   f (r , z )
, where f is a continuous function of r and z. Let E be the resulting electric field. Then the magnitude of
 E
(A) increases with r (B) is 0 (C) is 3 (D) decreases with z
Ans. (B)
Sol. Uniform charged ring is given it can be considered as static. A static electric charge produces an electric
field for which   E  0 i.e. conservation theorem states that equipotential surfaces have zero work
done on them, so whenever be apply stoke’s theorem on it clear that   E becomes zero proving that
E is static and irrotational
Hence correct option is (B)
Q.14 A soft-iron toroid is concentric with a long straight conductor carrying a direct current I. If the relative
permeability  r of soft-iron is 100, the ratio of the magnetic flux densities at two adjacent points located
just inside and just outside the toroid, is _______.
Ans. 100
Sol. Given :  r  100
A 2 dimensional view of the toroidal case is shown in the figure
Torque has magnetic field B  
Magnetic field density B at any point at a distance r is given by
r L
B
2r r
r–
 0 r I
Now just inside toroid B at r   wb/m 2
+
r
2r 
0 I
And just outside toroid B at r   
wb/m 2
2r
B at r 
 Required ratio   r  100 Ans.
B at r 
Q.15 RA and RB are the input resistances of circuits as shown below. The circuits extend infinitely in the
direction shown. Which one of the following statements is TRUE?
2W 2W 2W

RA 1W 1W 1W

2W 2W 2W

RB 1W 1W 1W 1W

(A) RA  RB (B) RA  RB  0 (C) RA  RB (D) RB  RA /(1  RA )


13 GATE 2016 [EE] Set - 1

Ans. (D)
Sol. Given :
2W 2W 2W 2W 2W

RA 1W 1W 1W RB 1W 1W 1W

Fig. (i) Fig. (ii)


By comparing figure (i) and (ii) we have

RB 1W RA

RB  1  RA
1  RA
RB 
1  RA
RA
 RB 
1  RA
Hence correct option is (D)
Q.16 In a constant V/f induction motor drive, the slip at the maximum torque
(A) is directly proportional to the synchronous speed.
(B) remains constant with respect to the synchronous speed.
(C) has an inverse relation with the synchronous speed.
(D) has no relation with the synchronous speed.
Ans. (C)
Sol. For any voltage and frequency
Slip corresponding to maximum torque in an induction motor is given by
R
S m  2 where X 2  2 fL2
X2
 1
Sm  
f N
And N f
v v
Now for constant   or variable   S m will be in inverse relation with frequency.
 f   f 
Therefore S m has inverse relation with synchronous speed as speed is directly proportional to frequency.
Hence the correct option is (C)
Q.17 In the portion of a circuit shown, if the heat generated in 5 Ω resistance is 10 calories per second, then
heat generated by the 4 Ω resistance, in calories per second, is _______.
4W 6W

5W
GATE 2016 [EE] Set - 1 14

Ans. (2 to 2)
Sol. We know that 1 calorie per second = 4.184 W Iy 4W 6W

Heat generated in 5 resistance  10 Cal/sec


 10  4.184  41.84 W
Also heat dissipated, I x2  5  41.84 Ix 5W

I x  2.89 A …. (i)
Voltage across resistance (5 ) and (4   6 ) should be equal, such that
I x  5  I y (4  6)
2.89  5
Iy 
10
 I y  1.44 A
 Heat generated by 4  resistance
 1.442  4  8.368 W
8.368
  2 calories/sec Ans.
4.184
Q.18 In the given circuit, the current supplied by the battery, in ampere, is _______.
I1 1W 1W I 2

1V 1W
I2

Ans. 0.5
Sol. Given :
1W N 1W

I1 I2

I2 1W
1V

Taking super mesh as I 2 is current source at boundary of two mesh


From figure applying KCL at node N
I1  I 2  I 2
I
 I2  1 …. (i)
2
Applying KVL to the super mesh and from equation (i), we get
I I
1V  I1  1  1  0
2 2
2 I1  1
1
I1   0.5 A Ans.
2
Current supplied by the battery in ampere in 0.5 A
15 GATE 2016 [EE] Set - 1

Q.19 In a 100 bus power system, there are 10 generators. In a particular iteration of Newton Raphson load
flow technique (in polar coordinates), two of the PV buses are converted to PQ type. In this iteration,
(A) the number of unknown voltage angles increases by two and the number of unknown voltage
magnitudes increases by two.
(B) the number of unknown voltage angles remains unchanged and the number of unknown voltage
magnitudes increases by two.
(C) the number of unknown voltage angles increases by two and the number of unknown voltage
magnitudes decreases by two.
(D) the number of unknown voltage angles remains unchanged and the number of unknown voltage
magnitudes decreases by two.
Ans. (B)
Sol. Total number of buses = 100
Generator bus  10  1  9
Load buses  100  10  90
Slack bus  1
It two of the PV buses are converted to PQ it will add 2 unknown voltages to iteration but unknown
angles remains constant.
Hence correct option is (B)
Q.20 The magnitude of three-phase fault currents at buses A and B of a power system are 10 pu and 8 pu,
respectively. Neglect all resistances in the system and consider the pre-fault system to be unloaded. The
pre-fault voltage at all buses in the system is 1.0 pu. The voltage magnitude at bus B during a three-
phase fault at bus A is 0.8 pu. The voltage magnitude at bus A during a three-phase fault at bus B, in pu,
is ________.
Ans. (0.84)
Sol. Post fault voltage magnitude at bus B during a 3   fault at bus A
VBA  0.8  VBB  Z AB I FA
 0.8  1  10  Z AB
 Z AB  0.02 p.u
Post fault voltage magnitude at bus A during a 3   fault at bus B,
VAA  VAB  Z AB I FB
VAA  1  0.02  8
 1  0.16  0.84 pu Ans.
Q.21 Consider a system consisting of a synchronous generator working at a lagging power factor, a
synchronous motor working at an overexcited condition and a directly grid-connected induction
generator. Consider capacitive VAr to be a source and inductive VAr to be a sink of reactive power.
Which one of the following statements is TRUE?
(A) Synchronous motor and synchronous generator are sources and induction generator is a sink of
reactive power.
(B) Synchronous motor and induction generator are sources and synchronous generator is a sink of
reactive power.
(C) Synchronous motor is a source and induction generator and synchronous generator are sinks of
reactive power.
(D) All are sources of reactive power.
GATE 2016 [EE] Set - 1 16

Ans. (A)
Sol. Given :
Synchronous generator working at a lagging power factor.
Synchronous motor working at an over excited condition
Induction generator directly connected to grid.
V - Curves
(1) For synchronous generator (2) For synchronous motor

Ia Ia

fy If
leading lagging lagging leading
under excited over excited under excited over excited
absorb P & Q deliver P & Q deliver P & Q absorb P & Q
P = Active power
Q = Reactive power
(3) Induction generator is sink of reactive power in inductive VAr .
Type of Machine Active Power (P) Reactive Power (Q)
1. Synchronous Generator Delivers Delivers or Absorbs *
2. Synchronous Motor Absorbs Delivers or Absorbs *
3. Induction Generator Delivers Absorbs
4. Induction Motor Absorbs Absorbs
 * indicates that delivers or absorbs depending upon control to the excitation systems according to the
system VAR requirement which facilitates flat voltage profile without any damage to the system due to
unbalanced reactive power.
Hence correct option is (A)
Q.22 A buck converter, as shown in Figure (a) below, is working in steady state. The output voltage and the
inductor current can be assumed to be ripple free. Figure (b) shows the inductor voltage vL during a
complete switching interval. Assuming all devices are ideal, the duty cycle of the buck converter is
________.
VL
M
VL 30 V

Vg D C V0 TON TOFF
R 0
t

- 20 V

TS

(a) (b)
17 GATE 2016 [EE] Set - 1

Ans. (0.4)
Sol. Given :
VL
M
VL 30 V

Vg D C V0 TON TOFF
R 0
t

- 20 V

TS

We know at steady state


Average voltage across inductor is zero
 Area under one cycle is zero
30 V  TON  20 V  TOFF  0
TOFF 30
 
TON 20
Adding one both side
TOFF 3
 1  1
TON 2
TOFF  TON 5
 
TON 2
TON 2
 
Ts 5
TON 2
Duty cycle    0.4 Ans.
Ts 5
Q.23 A steady dc current of 100 A is flowing through a power module (S, D) as shown in Figure (a). The V-I
characteristics of the IGBT (S) and the diode (D) are shown in Figures (b) and (c), respectively. The
conduction power loss in the power module (S, D), in watts, is ________.
I S (A) I D (A)

S D dV /dI = 0.02 W V0 = 0.7V dV /dI = 0.01 W


V0 = 1V

VS (Volt) VD (Volt)
100 A
V-I characteristic of IGBT V-I characteristic of diode
(a) (b)
GATE 2016 [EE] Set - 1 18

Ans. 169 to 171


Sol. From V - I characteristic of IGBT and diode
V0(diode) (i.e 0.7 V) V0(IGBT) (i.e 1V)
Hence only diode will work, current will flow through diode only and no current will flow through
IGBT .
Therefore conduction power loss in power module
PL  100  0.7  1002  0.01  170 W Ans.
Q.24 A 4-pole, lap-connected, separately excited dc motor is drawing a steady current of 40 A while running
at 600 rpm. A good approximation for the wave shape of the current in an armature conductor of the
motor is given by
(A) (B)
40 A
I
10 A I

t t

I 10 A
(C) I 10 A (D) T = 25 ms
T = 25 ms

t
t
-10 A T = 25 ms
T = 25 ms -10 A

Ans. (C)
Sol. Given : A 4 pole lap connected separately excited dc motor
N  600 rpm
Number of parallel path (A)  Number of poles (P)  4 ( LAP wound)
40
Current in each parallel path   10 A
4
Current in armature conductor  10 A
40 A

10 A 10 A 10 A 10 A

120 f 120  f
N (rpm)    600 rpm
P 4
f  20 kHz
1 1
Time period T    50 msec with straight line commutation
f 20
Hence the correct option is (C)
19 GATE 2016 [EE] Set - 1

Q.25 If an ideal transformer has an inductive load element at port 2 as shown in the figure below, the
equivalent inductance at port 1 is
n:1

Port 1 Port 2

2 n n2
(A) nL (B) n L (C) (D)
L L
Ans. (B)
N1
Sol. Given : n 
N2
n:1

Port 1 Port 2
Z 02
We know Z 01 
k2
N2
Where k 
N1

Referring port 2 inductance L to port 1 i.e. high voltage side,


The equivalent inductance at port 1 is n 2 L
Hence the correct option is (B)

Q.26 to Q.55 carry two marks each


Q.26 Candidates were asked to come to an interview with 3 pens each. Black, blue, green and red were the
permitted pen colours that the candidate could bring. The probability that a candidate comes with all 3
pens having the same colour is _____.
Ans. 0.01818
Sol. Given : Colours of pens Black, Blue, Green and Red
Probability of bringing 3 same colour pen
3C3  3C3  3C3  3C3 4
 
12C3 220

 0.01818 Ans.
GATE 2016 [EE] Set - 1 20


Q.27 Let S   n n where   1 . The value of  in the range 0    1 , such that S  2 is ______.
n0

Ans. 0.291 to 0.293



Sol. Given : S   n n
n0

S  0    2 2  33  ........
S    2 2  33  ........

S (sum of infinite series) {0    1 given}
(1  ) 2
Also S  2

 Equating sum  2
(1   ) 2
1
 (1  ) 2
2
1
1   
2
1 1
1   or 1   
2 2
   0.2928 or   1.707
But given range is 0    1
   0.2928 Ans.
Q.28 Let the eigenvalues of a 2 x 2 matrix A be 1, – 2 with eigenvectors x1 and x2 respectively. Then the
eigenvalues and eigenvectors of the matrix A2  3 A  4 I would, respectively, be
(A) 2,14; x1 , x2 (B) 2,14; x1  x2 , x1  x2
(C) 2, 0; x1 , x2 (D) 2, 0; x1  x2 , x1  x2
Ans. (A)
Sol. Given :
Eigen values of 2  2 matrix A be 1,  2
According to property of eigen values, we have
Eigen value of A2 is (1) 2 , ( 2) 2
i.e. 1, 4
Eigen value of  3A is ( 3) (1), ( 3) ( 2)
i.e.  3,6
Eigen value of 4 I is 4, 4
 Eigen value of A2  3 A  4 I is (1  3  4), (4  6  4) i.e. 2,14
Eigen would be x1 , x2
Hence the correct option is (A)
21 GATE 2016 [EE] Set - 1

Q.29 Let A be a 4 × 3 real matrix with rank 2. Which one of the following statement is TRUE?
(A) Rank of AT A is less than 2.
(B) Rank of AT A is equal to 2.
(C) Rank of AT A is greater than 2.
(D) Rank of AT A can be any number between 1 and 3.
Ans. (B)
Sol. Rank ( A43 )  2, ( A T34)  2
Hence (B)
Q.30 Consider the following asymptotic Bode magnitude plot (ω is in rad/s).
magnitude (dB)

12 dB
20 dB/dec - 40 dB/dec

0 dB
w
0.5 8

Which one of the following transfer functions is best represented by the above Bode magnitude plot?
2s 4(1  0.5s )
(A) (B)
(1  0.5s )(1  0.25s ) 2
s (1  0.25s )
2s 4s
(C) (D)
(1  2 s )(1  4 s ) (1  2 s )(1  4 s ) 2
Ans. (A)
Sol.
magnitude (dB)

12 dB
20 dB/dec - 40 dB/dec

0 dB w
w1 w2
0.5 8

From given bode plot corner frequencies are 1 rad/sec and 2 rad/sec
Initial slope is 20 dB/dec and therefore there is a zero at origin. At 1 slope change to 0 dB/dec,
 S 
signifies a factor of 1   in the denominator of the transfer function.
 1 
2
 s 
At 2 slope changes to – 40dB/dec that signifies a factor 1   in the denominator of the transfer
 2 
function.
GATE 2016 [EE] Set - 1 22

ks
Thus, T ( s )  2
 s  s 
1  1  
 1  2 
Now calculation of k, 1 and 2
From figure
0  20log k  20log 0.5
 k2 …. (ii)
 12(dB )  20log 2  20log 1
 1  2 rad/sec
Also 0  20log k1  40log8
 k1  64
12(dB )  20log 64  40log 2
 2  4 rad/sec
2s 2s
 T (s)  
 s  s 
2
(1  0.5s )(1  0.25s ) 2
1  1  
 2  4 
Hence the correct option is (A)
Q.31 Consider the following state-space representation of a linear time-invariant system.
1 0  1 1
x (t )    x(t ) , y (t )  cT x(t ) , c    and x(0)   
0 2  1 1
The value of y (t ) for t  log e 2 is ______.
Ans. (6 to 6)
Sol. Given :
1 0 
x (t )    x(t ) …. (i)
0 2 
Also we know x (t )  A x(t ) …. (ii)
On comparing equation (i) and (ii), we have
1 0 
A 
0 2 
Now y (t )  C T x(t )
x(t )  e At x(0)
Where e At  L1 [( SI  A) 1 ]
s 1 0 
 SI  A  
 0 s  2 
23 GATE 2016 [EE] Set - 1

 1 
 s 1 0 
 ( SI  A) 1   
 0 1 
 s  2 
 et 0
 e At   
0 e 2t 
 et 0  1  et 
And x(t )       
0 e 2t  1 e 2t 
 et 
 y (t )  1 1  2t   et  e 2t 
e 
Now for t  log e 2
y (t ) t log 2   eloge  e 2loge 
2 2

e  
 eloge  eloge   [2  4]  6
2 4
Ans.
 
s3
Q.32 Loop transfer function of a feedback system is G ( s ) H ( s )  . Take the Nyquist contour in the
s ( s  3)
2

clockwise direction. Then, the Nyquist plot of G(s)H (s) encircles 1 j0
(A) Once in clockwise direction (B) twice in clockwise direction
(C) Once in anticlockwise direction (D) twice in anticlockwise direction
Ans. (A)
s3
Sol. Given : G(s) H (s)  2
s ( s  3)
Characteristic equation
s3
1  G ( s) H ( s)  2 1  0 w = ¥+
s ( s  3) w = 0+
s 2 ( s  3)  s  3  0 w = 0-
w = ¥-
s 3  3s 2  s  3  0
Routh table
s3 1 1
s2 3 3
s1 2 0
s0 3
No. of sign change = 2
i.e. system is unstable, with two right half of the s-plane poles.
 Z  2, P  1
N  PZ
 1  2  1 which indicates encirclement once in the clockwise direction.
Hence the correct option is (A)
GATE 2016 [EE] Set - 1 24

Q.33 Given the following polynomial equation


s 3  5.5 s 2  8.5 s  3  0 ,
The number of roots of the polynominal, which have real parts strictly less than –1, is ______.
Ans. (2 to 2)
Sol. Solving polynomial for the roots,
s 3  5.5s 2  8.5s  3  0
s 3  3s 2  2.5s 2  7.5s  s  3  0
 s 2  ( s  3)  2.5s ( s  3)  ( s  3)  0
 ( s  3)( s 2  2.5s  1)  0
2.5  1.5
s   3,
2
1
s   3,  2,
2
So roots strictly less than 1 is  2,  3
Hence 2 is the correct answer
Q.34 Suppose x1 (t ) and x2 (t ) have the Fourier transforms as shown below.
X 1 ( jw) X 2 ( jw)

1 1
0.5 0.5
0.3 0.3

-1 0 1 2 w -2 -1 0 1 w

Which one of the following statements is TRUE?


(A) x1 (t ) and x2 (t ) are complex and x1 (t ) x2 (t ) is also complex with nonzero part
(B) x1 (t ) and x2 (t ) are real and x1 (t ) x2 (t ) is also real
(C) x1 (t ) and x2 (t ) are complex but x1 (t ) x2 (t ) is real
(D) x1 (t ) and x2 (t ) are imaginary but x1 (t ) x2 (t ) is real
Ans. (*)

Q.35 The output of a continuous-time, linear time-invariant system is denoted by T {x(t )} where x(t ) is the
input signal. A signal z (t ) is called eigen-signal of the system T , when T {z (t )}   z (t ) , where  is a
complex number, in general, and is called an eigenvalue of T. Suppose the impulse response of the
system T is real and even. Which of the following statements is TRUE?
(A) cos (t ) is an eigen-signal but sin (t ) is not
(B) cos (t ) and sin (t ) are both eigen-signals but with different eigenvalues
(C) sin (t ) is an eigen-signal but cos (t ) is not
(D) cos (t ) and sin (t ) are both eigen-signals with identical eigenvalues
25 GATE 2016 [EE] Set - 1

Ans. (D)
Sol. Given : Impulse response is real and even
Thus H ( j0 ) will also be real and even
Therefore H ( j0 )  H ( j0 )

eit  eit
Now cos(t ) is input i.e. is input
2
H ( j1)eit  H ( j1) e  it  eit  e  it 
Output will be  H ( j1)  
2  2 
 H ( j1)  cos(t )
If sin(t) is input i.e.
eit  e  it
is input
2
H ( j1)eit  H ( j1)e  it eit  e  it
Output will be  H ( j1)
2 2
So sin(t) and cos(t) are eigen signal with same eigen values.
Hence correct option is (D)
Q.36 The current state QA QB of a two JK flip-flop system is 00. Assume that the clock rise-time is much
smaller than the delay of the JK flip-flop. The next state of the system is
5V
QA QB
J J

QA
K K

CLK
(A) 00 (B) 01 (C) 11 (D) 10
Ans. (C)
Sol. Given : Synchronous counter
When clock is applied both flip flops will change their state simultaneously depending upon JK flip flop.
State table :
PS PI NS
QA QB JA KA J B (QA ) K B (QA ) QA QB
1 1
0 0 1 1 1 1 1 1

The next state of the system QAQB  11


Hence the correct option in (C)
GATE 2016 [EE] Set - 1 26

Q.37 A 2-bit flash Analog to Digital Converter (ADC) is given below. The input is 0  VIN  3 Volts. The
expression for the LSB of the output B0 as a Boolean function of X2, X1, and X0 is
3V

100 W
X2

200 W
B1
X1 Digital
Circuit
B0
200 W

X0

100 W

VIN

(A) X 0 [ X 2  X 1 ] (B) X 0 [ X 2  X 1 ] (C) X 0 [ X 2  X 1 ] (D) X 0 [ X 2  X 1 ]


Ans. (A)
Sol. Given :
3V
Comparator
100 W

2.5V X2

200 W
B1
1.5V X1 Digital
Circuit
B0
200 W

X0
0.5V

100 W

VIN
0 £ VIN £ 3
Vin (v) X 2 X 1 X 0 B1 B0
0 0 0 0 0 0
1 0 0 1 0 1
2 0 1 1 1 0
3 1 1 1 1 1
B0  X 2 X 1 X 0  X 2 X 1 X 0
 ( X 2 X1  X 2 X1) X 0  ( X 2  X1) X 0
B0  X 0  X 2  X 1 
Hence correct option is (A)
27 GATE 2016 [EE] Set - 1

Q.38 Two electric charges q and −2q are placed at (0,0) and (6,0) on the x-y plane. The equation of the zero
equipotential curve in the x-y plane is
(A) x  2 (B) y  2 (C) x 2  y 2  2 (D) ( x  2) 2  y 2  16
Ans. (D)
Sol. Given : Charge q is located at (0,0) and −2q is located at (6,0)
To find : V at any point ( x, y ) y
q
Vq 
4 x 2  y 2
 2q V0 ( x, y )
V2 q 
4 x  y
2 2
x
q  2q q (0,0) - 2q (6,0)
Vtotal  0   0
4 x 2  y 2 4 ( x  6) 2  y 2

( x  6) 2  y 2  2  x2  y 2 
 x 2  36  12 x  y 2  4 x 2  4 y 2
 3 x 2  3 y 2  12 x  36
 x 2  y 2  4 x  12
 x 2  4 x  4  y 2  16  ( x  2) 2  y 2  16
Hence the correct option is (D)
Q.39 In the circuit shown, switch S2 has been closed for a long time. At time t = 0 switch S1 is closed. At t =
0+, the rate of change of current through the inductor, in amperes per second, is _____.
S1 1W S2

2W
3V 1H
3V

Ans. (2 to 2)
Sol.
S1 1W S2

2W
1H
3V

At time t  0 , network is in steady state with switch S1 open and S 2 closed


GATE 2016 [EE] Set - 1 28

1W

2W
3V iL (0- )
3V

3
So, iL (0 )   1.5A
2
At time t  0
1W A

2W
3V 1H
3V

KCL at node A.
VA  3 3 VA  3
  0
1 2 2
2(VA  3)  3  (VA  3)  0
3VA  6
VA  6
VA  2
di (0 )
VA  L 2
dt
di (0 ) 2 2
    2 A/sec Ans.
dt L 1
Q.40 A three-phase cable is supplying 800 kW and 600 kVAr to an inductive load. It is intended to supply an
additional resistive load of 100 kW through the same cable without increasing the heat dissipation in the
cable, by providing a three-phase bank of capacitors connected in star across the load. Given the line
voltage is 3.3 kV, 50 Hz, the capacitance per phase of the bank, expressed in microfarads, is _______.
Ans. (48 to 48)
Sol. Given : A -3 supply 800 kW and 600 kVAr
Case - I
kVA1  8002  6002  1000 kVA
Current should be constant for constant heat dissipation
Case : II
Active power  P2  800  100  900 kW

Reactive power Q2  10002  9002  435.889 kVAr


Reactive power supplied by 3- bank  600  435.889  164.11 kVAr
29 GATE 2016 [EE] Set - 1

Qbank 164.11
  54.7 kVAr
ph 3

V 3.3
  1.9052 kV
ph 3
2
V 
 
Qc  ph 

ph XC

(1.9052 103 ) 2
XC   66.36 
54.7  103
1 1
C   47.96 F Ans.
2  f X C 2 50  66.36

Q.41 A 30 MVA, 3-phase, 50 Hz, 13.8 kV, star-connected synchronous generator has positive, negative and
zero sequence reactances, 15%, 15% and 5% respectively. A reactance (Xn) is connected between the
neutral of the generator and ground. A double line to ground fault takes place involving phases ‘b’ and
‘c’, with a fault impedance of j0.1 p.u. The value of Xn (in p.u.) that will limit the positive sequence
generator current to 4270 A is _________.
Ans. (1.07)

30 103
Sol. Base current I B   1255.109 A
3 13.8

Fault current I f  4270 A

4270
I pu   3.402 pu
1255.109
 For double line to ground fault
Ea
I g1 
X1  ( X 2  X 0 )

Where X 0  X 0  3( X n  X f )

X 0  3 X n  0.35

1.0
3.402 
0.15  (3 X n  0.35)
0.15 
0.15  3 X n  0.35

Solving above equation we get


Z n  1.07 pu Ans.
GATE 2016 [EE] Set - 1 30

Q.42 If the star side of the star-delta transformer shown in the figure is excited by a negative sequence
voltage, then
A a

B N

c
C b

(A) VAB leads Vab by 600 (B) VAB lags Vab by 600

(C) VAB leads Vab by 300 (D) VAB lags Vab by 300
Ans. (D)
Sol. Given :
+A a
+
V Ð00 + -
3V Ð - 300
- EÐ00 EÐ -1200
-B N
- - +
+ -
V Ð1200 V Ð - 1200
c
+ + EÐ1200
-
C b

With negative sequence voltage


VAN  V 00 , VBN  V 1200

VCN  V  1200 VAN = V Ð0


VAB  3V   300 i.e. VAN  VBN 300
-VBN
Also Vab  E0 0
VAB

 VAB lags Vab by 300


Hence the correct option is (D)
Q.43 A single-phase thyristor-bridge rectifier is fed from a 230 V, 50 Hz, single-phase AC mains. If it is
delivering a constant DC current of 10 A, at firing angle of 300 , then value of the power factor at AC
mains is
(A) 0.87 (B) 0.9 (C) 0.78 (D) 0.45
Ans. (C)
Sol. For a single phase bridge rectifier
2 2
Power factor at input mains  cos 

 0.9cos300
 0.78
Hence the correct option is (C)
31 GATE 2016 [EE] Set - 1

Q.44 The switches T1 and T2 in figure (a) are switched in a complementary fashion with sinusoidal pulse
width modulation technique. The modulating voltage vm (t )  0.8sin(200t )V and the triangular carrier
voltage ( vc ) are as shown in figure (b). The carrier frequency is 5 kHz. The peak value of the 100 Hz
component of the load current (i1), in ampere, is ___________.
vc
T1
Vdc /2 = 250 V
iL 1
0.8

t
X L = 16 W at R = 12 W
100 Hz T2
vm

Vdc /2 = 250 V

(a) (b)
Ans. (10)
Sol. Given : vm (t )  0.8sin (200t ) V
Modulation index
ma  0.8
Amplitude of fundamental output voltage
V
(V01 ) peak  ma  d
2
 0.8  250  200 V
Peak value of the 100 Hz component of the load current with   200 
(V )
( I 01 ) peak  01 peak
4
200 200
 
R  (XL)
2 2
R  ( L ) 2
2

200
 2  10 A Ans.
12  162
Q.45 The voltage ( vs ) across and the current ( is ) through a semiconductor during a turn-ON transition are
shown in figure. The energy dissipated during the turn-ON transition, in mJ, is _________.

vs 600 V

0
t
50 V
is
100 A
0
t
T 1 = 1ms T 2 = 1ms
GATE 2016 [EE] Set - 1 32

Ans. (75 to 75)


T1 T1

Sol. Energy loss during T1   v.i dt  600   i dt


0 0

 600  Area under T1


1
 600  150 110 6  45 mJ
2
T2 T2

Energy loss during T2   v.i dt  100   v dt


0 0

 100  Area under T2


1
 100   600 10 6  1  30 mJ
2
Total energy loss  45  30  75 mJ Ans.
Q.46 A single-phase 400 V, 50 Hz transformer has an iron loss of 5000 W at the rated condition. When
operated at 200 V, 25 Hz, the iron loss is 2000 W. When operated at 416 V, 52 Hz, the value of the
hysteresis loss divided by the eddy current loss is __________.
Ans. (1.44)
v 400
Sol. Given : At 400 V, 50 Hz , pi  5000 W,  8
f 50
v 200
At 200 V, 25 Hz , pi  2000 W,  8
f 25
  v 
Now iron loss pi  af  bf 2  At const   
  f 
 5000  a  50  b  502 …. (i)
2000  a  25  b  252 …. (ii)
Solving equation (i) and (ii) we get
a  60
4
b
5
v 416
For 416 V,52 Hz,  8
f 52
ph  52  60  3120
4
pe  522   2163.2
5
ph 3120
 Ratio    1.44 Ans.
pe 2163.2
33 GATE 2016 [EE] Set - 1

Q.47 A DC shunt generator delivers 45 at a terminal voltage of 220 V. The armature and the shunt field
resistances are 0.1 and 44  respectively. The stray losses are 375 W. The percentage efficiency of
the DC generator is _____________.
Ans. (86.84)
Sol.
5A 45A = I 0
50 A +
Ra 0.01W
Rsh 44 W 220 V

 Total copper losses  52  44  502  0.01


 1125 W
Stray losses  375 W
Total loss  1125  375  1500 W
Output  220  45  9900 W
9900
Efficiency   100  86.84% Ans.
9900  1500
Q.48 A three-phase, 50 Hz salient-pole synchronous motor has a per-phase direct-axis reactance ( X d ) of 0.8
pu and per-phase quadrature axis reactance ( X q ) of .6 pu. Resistance of the machine is negligible. It
drawing full current at 0.8 pf (leading). When the terminal voltage is 1 pu, per-phase induced voltage, in
pu, is ___________.
Ans. (1.606)
Sol. Given : Synchronous motor is at leading pf
X d  0.8 p.u,   36.860 ( pf is 0.8 leading)
X q  0.6 p.u, Rq  0
V sin   I a  Rq
tan   …. (i)
V cos 
Equating above values in equation (i), we get
  56.300
We know   
E
   19.70 0

Now E  V cos   I d X d d jI A X s

I d  I a sin   0.831 q IA I A RA
V
 E  (1) cos19.7  (0.831)(0.8)
E  1.606 p.u Ans.
GATE 2016 [EE] Set - 1 34

Q.49 A single-phase 22 kVA, 2200 V/220 V, 50 Hz, distribution transformer is to be connected as an auto-
transformer to get an output voltage of 2420 V. Its maximum kVA rating as an auto-transformer is
(A) 22 (B) 24.2 (C) 242 (D) 2420
Ans. (C)
Sol. Given : A single phase 22 kVA, 2200 V/220 V, 50 Hz
22  103
Rated lv current   100 A
220
22  103
Rated hv current   10 A
2200
10 A 2200/220 V
A C
100 A

B D
Auto transformer to get output 2420 V
C
+ 100 A

220 V

110 A D - 2420 V
A
10 A +

2000 V

-
B
Maximum kVA rating  2420  100  242 kVA
Hence the correct option (C)
Q.50 A single-phase full-bridge voltage source inverter (VSI) is fed from a 300 V battery. A pulse of 1200
duration is used trigger the appropriate devices in each half-cycle. The rms value of the fundamental
component of the output voltage, in volts, is
(A) 234 (B) 245 (C) 300 (D) 331
Ans. (A)
Sol. For single phase full bridge VSI
2 2
(V01 ) max  Vs sin d

Pulse width 2d  1200
d  600
2 2 3
 (V01 ) rms   300   234 V Ans.
 2
Hence the correct option (A)
35 GATE 2016 [EE] Set - 1

Q.51 A single-phase transmission line has two conductors each of 10 mm radius. These are fixed at a center-
to-center distance of 1 m in a horizontal plane. This is now converted to a three-phase transmission line
by introducing a third conductor of the same radius. This conductor is fixed at an equal distance D from
the two single-phase conductors. The three-phase line is fully transposed. The positive sequence
inductance per inductance per phase of the three-phase system is to be 15% more than that of the
inductance per conductor of the single-phase system. The distance D, in meters, is ______.
Ans. (1.438)
Sol. For single phase
1m

10 mm

For three phase


1m

D D

1.05  L1-   L3- 


1000 (1000 D 2 )1/3
1.05  0.2ln  0.2ln
0.788 10 0.788 10
1.05
 1000  (1000 D 2 )1/3
  
 7.88  7.88
D  1438 mm  1.438m Ans.
Q.52 In the circuit shown below, the supply voltage is 10 sin(1000t) volts. The peak of the steady state current
through the 1 resistor, in amperes, is _______.
2 mF
4W

250 mF 1W 500 mH
5W

4 mH

10sin(1000 t)
GATE 2016 [EE] Set - 1 36

Ans. 1A
Sol. Given :
- j 500W
4W

- j 4W 1W + j 500 W

5W
+ j 4W

10sin(1000 t)
Input = 10sin(1000 t ) volts
At   1000 rad/sec parallel LC circuit will offer infinite resistance open circuit
4W
o.c
1W
5W
o.c

10sin(1000 t)
10sin1000 t
I  sin1000 t , peak current = 1 A Ans.
4 1 5
Q.53 A dc voltage with ripple is given by v(t )  [100  10sin(t )  5sin(3t )] volts. Measurement of this
voltage v(t ) , made by moving-coil and moving-iron voltmeters, show readings of V1 and V2
respectively. The value of V2  V1 , in volts, is ________.
Ans. 0.31
Sol. Given : v(t )  100  10sin t  5sin 3t
V1  Reading by moving coil = average or dc value  100 V
V2  Reading by moving iron  Rms value
2 2
 10   5 
 100  
2
    100.31V
 2  2
 V2  V1  100.31  100  0.31V Ans.
Q.54 The circuit below is excited by a sinusoidal source. The value of R in  , for which the admittance of
the circuit becomes a pure conductance at all frequencies is __________.
100 mF R

0.02 H R

Vs
37 GATE 2016 [EE] Set - 1

Ans. 14.14 to 14.14


Sol. Taking identical time constant, l  c
L
 RC
R
L
R2 
C
L
 R
C
For above circuit value of R in 
L 0.02
R   200  14.14  Ans.
C 100 10 6
Q.55 In the circuit shown below, the node voltage VA is _____________V.
A I1 5W

5W 5W
5W 5A
10I1 10 V

Ans. (11.42)
Sol. Given :
A I1 5W

5W 5W
5W 5A
10I1 10 V

From figure it is clear that,


V  10 VA  10
I1  A  …. (i)
55 10
Now applying KCL at A
VA VA  (10 I1 ) V  10
 5 A 0
5 5 10
From equation (i), we get
VA VA (V  10) VA  10
 2 A   50  0
5 5 10 10
2VA  2VA  2(VA  10)  (VA  10)  50  0
7 VA  30  50
80
VA   11.42 Volts Ans.
7



You might also like